Conference Notes 7-3-2014

Here's to the start of another outstanding academic year!  Welcome to all the new ACMC EM Interns!

 Beckemeyer/Girzadas  Oral Boards

Case1.  30 yo pregnant female presented after MVC.  Patient had hypotension, abdominal pain/cramping and vaginal bleeding. FAST showed no intra-peritoneal blood and the fetus was bradycardic. Traumatic Placental Abruption with associated DIC was the diagnosis.  Critical actions were to place patient in left lateral decubitus position to off-load the IVC, resuscitation with blood products and fluids, consult OB to perform emergent c-section.   One learning point was that in this situation it would be prudent to activate the massive transfusion policy.  These patients may require large quantities of PRBC’s, platelets, FFP and cryoprecipitate.  Another key point is that ultrasound is insensitive for diagnosing placental abruption.  However, if ultrasound is positive for abruption, it is very specific.  In the stable patient, fetal monitoring has a higher sensitivity for picking up abruption.

 Case2.  40 yo male presents with palpitations and rapid heart rate.  EKG showed an irregular rhythm with both wide and narrow complexes. 

Diagnosis was AFIB with WPW aberrant conduction.  

Critical actions:  attempt chemical cardioversion with procainamide (preferred choice), ibutilide, or amiodarone.  When the patient decompensated and became hypotensive, proceed to electrical cardioversion.  Harwood made the point that Afib can be resistant to electrical cardioversion so it was prudent to perform cardioversion at an initially high dose like 150-200J.

Case3.   6 yo female with gross deformity of her left elbow after falling from her bike.  Xrays showed a Gartland 3 supracondylar fracture.  Critical actions were to treat pain, perform a careful NV exam, and arrange urgent/emergent surgery.  There was general agreement among the faculty that they would not attempt closed reduction of a displaced supracondylar fracture unless the neurovascular status of the affected limb was severely impaired and there was going to be a significant delay to get the patient to the OR.   If the patient has a diminished radial pulse, check for warmth of fingers, cap refill, pulse ox wave form of fingers of affected extremity, and doppler the radial pulse.   If any of these are present and you can get the patient to surgery within 2-4 hours just keep patient in the position of comfort and no need for the emergency physician to attempt closed reduction.

 

Lovell/Urumov/C. Kulstad     How to Give a Good Lecure

First tip: To give a more engaging lecture minimize your use of power point.   Try to think of non-powerpoint methods to interact with the audience.

Be sure you have your technology pre-tested,  and make sure it is working before you try to use it during a presentation.

Elise gave a “bad presentation” example and the audience made the following comments:  You need to have good contrast between fonts/background on your slides.   Spell check your slides.   Get out from the podium to interact with the audience.    Change up your intonation and pace of speech to keep the audience engaged.  Limit your slides to 6 lines with only 6 words in each line.   More than that is too much.  Reading your  slides verbatim is the marker of not being prepared/putting the talk together at the last minute. 

 The audience is only capable of picking up 3 key points from any lecture.  So hammer home those points.

 Elise’s “good talk” example was given next:  Talk had an exciting title that immediately hooked the audience.  She asked the audience questions.  Each topic started with a relevant case example.  She gave focused learning points.  The lecture points were very EM clinically oriented.  She hammered home the key points repeatedly.  More useful diagrams on her slides.  Slides had no more than 6 lines with 6 or less words in each line.  She knew her audience and really presented the topic in an EM relevant way.   She wrapped up the talk with a nice summary/wrap-up/repeat of the key points.    Harwood comment:  Powerpoint is not all bad.  It helps you organize your topic.  It allows you to use pictures and videos and give info in a concise way.  The key is to be interactive when using powerpoint.

 Andrej’s “bad talk”:   Audience comment:  Anrdrej was apologizing for his talk.  Multiple spelling errors on his slide.   Too much data on a slide.   He had a phone call interruption during his talk.   He was very non-specific in his learning points.  Poorly developed explanations of the information on his slides.   He started reading his slides facing away from his audience and toward the screen.   Impossible to read his slides due to lack of contrast.  Unappealing slides.  Distracting transitions.   Weak opening.  Self-undercutting of his authority.  Playing with coins in his topic was very distracting.

 Iannitelli comment: About 10% of the population is color bind so black/white and blue/yellow or blue/white are better than most other colors.

Andrej’s “good talk”:  Much more interesting title.  Slide colors were very good.  He engaged the audience and asked them questions and examples.  Kept slides to six lines with no more than 6 words per line.  Graphs and charts were much more clear and he explained them with much more clarity.

 Christine’s  “bad talk”:  Too much info on the slides.  Pictures on the slides were very small.   She was shifting back and forth constantly while facing the screen.   She frequently contradicted herself while speaking.   There were misspellings on the slide.  Her video link did not work.    

Christine’s “good talk”: Much more interesting title.   Pictures larger and more pertinent to her presentation. Much fewer words on each slide.   She faced the audience while speaking.  Not constantly shifting back and forth.   The explanation of each slide was much more clear.   The intonation and pace of her speech was much more varied and effective.

 Elise comment: Always finish your talk early to be respectful of peoples’ time.

 

 Beckemeyer and Permar     Resuscitation of the Critically Ill/Cardiac Arrest Patient

The ABCDEF's of critical thinking during a resuscitation. 

 A=Aorta and acidosis.  Always consider these two entities as causes of shock.

B=Bagging and Baby on Board.   Avoid hyperventilating patients and always consider that a female may be pregnant

C=Compressions and Cooling.  Push hard and fast for CPR and Cool the patient when you get ROSC.

D= Defibrillation/CPR push hard and push fast/Cool the patient if you get ROSC.  Keep compressions going when you are charging the defibrillator.   There is some debate about the safety of compressions during defibrillation. Harwood comment: it is probably safe.  Erik comment:  there is no way for the person doing compressions to sustain a shock from the defibrillator.

D=Drugs/Dextrose   Consider overdose and hypoglycemia in the altered critically ill patient.  Multiple attendings said they had intubated patients who were later found to be hypoglycemic.

D=DOPES  mnemonic for ventilation/oxygenation problems with ventilator.  Dislodgement/Obstruction/Pneumothorax/Equipment failure/Stacking breaths in COPD/Asthma

E=Effusion and Embolus   Pericardial effusion can look clinically like a PE.   Do an echo before you start treating PE.  The treatment for these two diseases is markedly different.   Electrical alternans is classically associated with pericardial effusion.  Echo can be helpful identifying both diseases.  EKG in PE can show S1Q3T3 and/or t wave inversion inferiorly and anteriorly. 

If BP improves markedly with intubation consider the diagnosis of PE. Positive pressure ventilation improves the hemodynamics of the PE patient.  

F=Forget about it.   These don’t work for resuscitation: Bicarb, mag, lidocaine, and atropine.  Lidocaine and defasiculating meds for RSI.   Trandelenburg positioning.

 

LVAD  Team      Care of the LVAD Patient

 Most patients would survive for awhile if the LVAD battery power ran out.  Some would die in a few minutes.

All patient who get an LVAD have an EF <25%.  They are all in class 3-4 heart failure.

LVAD has been shown to improve quality of life and survival.  The LVAD team has a patient who is still working as an anesthesiologist with an LVAD!  

All VAD’s have the same 4 components: 1. Inflow cannula in the apex of left ventricle pointing at the mitral valve.   2. Mechanical pump.    3. Outflow cannula to the aorta.   4. Drive line.   The drive line is connected to a controller.    The LVAD team has seen multiple patients present through the ED with a disconnected drive line.  This simple problem can prove to be fatal.  

 If a patient presents with critically low battery power.  Call SHU or 9E/W to get extra batteries brought down.

Everyone in the audience was given the opportunity  to connect/disconnect the drive line and battery packs to practice.  If a patient is in the ER with a disconnected drive line or battery, these need to be re-connected ASAP.   Some patients have died due to a disconnected drive line.

 There should be a VAD Coordinator in the hospital from 7a to 7p.   If they are not physically present in the ED they are available by phone.  ASHU is another resource to contact if you have any difficulty getting a VAD Coordinator.

 Resucitation:  Chest compressions are indicated during resuscitation if the pump is not working (If the pump is not working, it will be alarming).  If the pump is working  and not alarming, you don’t need to do CPR because the pump is working.  

It is thought that good chest  compressions prevent clotting of an LVAD with pump failure.  Defibrillation is OK for V-fib.  Never disconnect patient from LVAD equipment.   If the pump is not working it is ok to give milrinone to increase contractility.   It is ok to give levophed for hypotension.

 If  AICD is firing,  get the AICD interrogated.  There is a phenomenon of “phantom shocks” in patients who have been shocked multiple times in the past.  Consult the EP cardiologist.  The LVAD team fixes the plumbing (LVAD) but doesn’t deal with the electricity (AICD).   The most common problem causing AICD firing is hypovolemia.   Dehydration from the flu is enough to cause hypovolemia and AICD firing.  

Depending on how high the flow rate is set on the LVAD, a patient may or may not have a pulse.  Accurate measurement of  BP is only possible via an A-line in a patient with no pulse.  You may be able to hear a single kortokof sound when taking a brachial bood pressure using a Doppler.  In a pulseless patient that sound corresponds most likely to a mean arterial pressure.  In a patient with a pulse the kortofkoff sound corresponds to a systolic pressure.  

It is ok to give cautious fluids to a patient you think is hypovolemic.   Echo may be helpful to determine if a patient is hypovolemic.  You may see LV collapse due to the pump sucking all the blood out of the under-filled LV.

If the VAD is not alarming, the VAD is working.  In an altered patient with a non-alarming LVAD, the patient’s altered mental status is likely due to a non-VAD cause.

Elise comment:  In a hypotensive LVAD patient  that does not have a pump alarm going off  you have got to look for non-VAD causes.   LVAD team response: Get CT head for stroke, check their HGB (significant anemia for these patients is around 7-8), do a rectal exam for GI bleeding, look for sepsis,  check an LDH and peripheral smear for hemolysis. 

 There is one case of pump failure that won’t cause the pump to alarm and that is a clotted pump.  The pump doesn’t alarm because it is still running.  However it can’t pump clotted blood. These patients will present with chest pain, SOB, acute heart failure, and hemolysis (LDH > 1000).   These patients need to go to the OR immediately for a new pump. 

 Best central line approach in the LVAD patient is a Right IJ.

 

Conference Notes 6-11-2014

Farewell to the Class of 2014!  You are an awesome class and we are going to miss you! (See pic below)

No conference notes for the next two weeks: Pig Lab next week and Intern Orientation starts the following week.  We'll pick back up with conference on July 2nd. 

Kmetuk/Bolton      Oral Boards

 

Case 1.  Infant presents with Jaundice.  Total bili is 22 with indirect bili predominating.   Diagnosis was breast feeding jaundice. Critical actions: Get rectal temp, order CBC/coombs/smear/fractionated bilirubin, administer IV fluids, initiate phototherapy.   Most common jaundice is physiologic.  Always be alert for sepsis causing jaundice in an infant(Look for fever, poor feeding, not well appearing).  Infants can also have breast feeding or breast milk jaundice.    Elise comment: I don’t routinely get a coombs test on jaundiced infants, unless there are signs of hemolysis on the CBC or the patient appears ill. Harwood comment: you gotta plot out the bilirubin level on the time graph.

 

*Bilirubin/Time Chart for Phototherapy

Case 2.  Patient presents with pulseless/painful leg.  Pt was also found to have afib.  Critical actions:  Give pain medication, start IV heparin, consult vascular surgery.   If you get an ABI to evaluate arterial flow in a limb, any measurement less than 0.9 is a concern.  Less than 0.5 mandates urgent vascular consult.  Girzadas comment: If you identify a pulseless extremity,  think Afib, dissection, and endocarditis in addition to severe atherosclerosis.  Harwwod comment: You have to check every pulse in the affected extremity to try to identify where the clot is.

Case 3. 18yo male with SOB.   Patient was found to be cyanotic.   Methemoglobin level was elevated to 55%.  Pulse ox was 85%.  Further history identified that the patient was using “poppers”   Critical actions:  Intubation, get ABG with co-ox/metHgb, administer IV methylene blue (1-2mg/kg).  MetHGB can be measured on VBG.  Patients with G6PD don’t respond to methylene blue.   Poppers are amyl nitrate inhalers and are used for getting high and for sphincter relaxation.   Girzadas comment: Andrea and Ted always make the point that if you see a pulse ox of 85% you gotta think methemoglobinemia.  Andrea comment: Methemoglobin produces a light wavelength that gives you a pulse ox of 85%.   Andrea comment: Don’t give narcan to an intubated patient. It’s just poor form.  You have the airway managed so narcan will do nothing for the patient except possibly agitate them. 

  Elise comment: Check pupils and skin to identify a toxidrome in any tox case on oral boards.  

 

Hemming   Penetrating Neck Injuries

 Zone 1 of the neck:  Includes lungs, aorta, subclavian vessels, vertebral arteries, esophagus

Zone 2: cricoid to angle of mandible: Largest/Most commonly injured area

Zone 3: Angle of the mandible to base of the skull.   Difficult surgical access. 

Girzadas comment: To remember if zone 3 is at the top or the bottom, think 3rd ventricle.  There is not 3 of anything in the chest .  Humerous Elise comment: There are 4 ventricles though!   Maletich comment: I think of an elevator going up to the third floor.

* Zones of neck with structures in each zone 

Hard signs of significant neck injury: active external bleeding, bruit or thrill, expanding/pulsatile hematoma, oropharyngeal bleeding, sucking neck wound, neuro deficit.  subQ air, blood in oropharynx, stridor.

 Traditional approach:  If there is  physical sign of penetrating injury to neck:

 zones 1&3 if the patient is stable, should get CT angiography/endoscopy/

bronchoscopy.

zone 2 should go to OR for exploration (stable or unstable).

There is a “no zone” approach:  If penetration of any zone and stable, then patient gets a CT angio +/_ esophogoscopy/bronchoscopy.  If significant injury found then patients goes to OR .  Unstable patients go to OR without imagng.

 Flexible endoscopy has very high sensitivity for esophageal injuries.  CT can miss some esophageal injuries.

1/3 of vertebral artery injuries are asymptomatic.  Most do not have neurologic sequelae. 

Zone 2 injuries with physical findings require exploration.

RSI is considered safe

Physical exam is a powerful tool to identify significant neck injuries.

Elise comment: Is trauma taking all Zone 2 injuries to OR?  Hemming response: If patient has no hard signs of injury and a non-threatening bullet trajectory, they will do CT/endoscopy.

Harwood comment: If I was working in a non-level 1 hospital and was confronted with a patient who had suffered a penetrating neck injury, my general approach would be  to first intubate those who need it .  Next, any Zone 2 injury gets trasnferred to the nearest Trauma Center.   Zones1 & 3 with low risk signs symptoms can be managed selectively with CT Angio and possibly not get transferred.  This raised a lively discussion about who to transfer.  The majority of faculty would transfer all penetrating neck injuries and not try to manage any of them at a non-level 1 ED.

 

Carlson    Toxicology Axioms

16 yo female with epigastric pain. There was family conflict preceding this episode of pain.  Pt had  prior history of cutting and overdose.  Labs show hyperglycemia and acidosis.  ASA is undetectable.  On further lab investigation, Iron level was 800.  If you see hyperglycemia and metabolic acidosis think ASA and Iron.    Axiom: It’s not tox until you think of tox.

Tox Mimicry: It looks like Food poisoning: could be CO, digoxin, arsenic.   You suspect SubArachnoid Hemorhage: think CO.   You are considering Meningitis:  Think ASA.    Ibuprofen overdose can also give meningeal signs.  Looks like Sepsis: think cyanide, calcium channel blockers, asa.    You are pretty sure it is DKA: consider Iron, AKA, ASA, toxic alcohols, CCB’s, cyanide.  Pt has Acute psychosis: consider hallucinogens, sympathomimetics, salicylates, digoxin, anticholinergics, alcohol/sedative withdrawl.

 46 yo male with hx of MS and depression. Acutely confused.  Family suspects baclofen overdose because they can’t  find the baclofen bottle.  Pt was febrile and hypertensive.  Pt had abdominal pain and diarrhea.  This is a classic picture of baclofen withdrawl not baclofen overdose.  Pt had placed the plastic jar containing all his baclofen in his rectum for no logical reason.  So he did indeed have baclofen withdrawl.  All his baclofen was in the plastic bottle that was shoved in his rectum.

 Male in respiratory distress after using “cocaine”.  Patient clinically has SLUDGE syndrome.  Cocaine does not normally cause a SLUDGE toxidrome, it causes a sympathomimetic toxidrome.   The Cocaine was cut with malathion (organophosphate insecticide) causing SLUDGE.   Axiom: Know your Toxidromes.   Opioid, sympathomimetic, sedative-hypnotic, anticholinergic, cholinergic, opioid withdrawl.    Kelly comment: we had a case recently of a patient with an opiod appearing toxidrome that turned out to be a pontine hemorrhage.  Kind of a reverse tox mimic.

 Axiom: Check your Ego at the door. Don’t be shy, call the poison center for help.

 3yo male found with open bottle of Tylenol.   PMD sent child to ED to get NG tube and activated charcoal.  ED doc should always calculate the worse case scenario based on type of medication and the size of pill bottle.  Check APAP level at 4 hours.  No need for NG and charcoal.  If 4 hour level is toxic, start  NAC.  If it is non-toxic, patient can be discharged.  Axiom: Sometimes it is OK to do nothing. It is OK not to decontaminate. Just document that the risks of decontamination outweigh the benefit.  Decontamination options are gastric lavage, activated charcoal, and whole bowel irrigation.   The senior faculty had a chuckle about how times have changed when a quick survey showed that none of the current residents had ever done a  gastric lavage on a patient.     Old school  faculty reminisced that we all did probably 3 lavages a weekend when we were residents.  Definitely the emergency management of toxic ingestions has evolved significantly over the years.  All faculty were glad that we don’t have to lavage patients hardly ever anymore.

 

44yo female who overdosed on INH.  Antidote is pyridoxine gram for gram.  Axiom: Don’t get  caught with your scrubs down.  As soon as you start to consider this diagnosis, call the pharmacy to start mobilizing enough pyridoxine. You might need to call other hospitals. Pyridoxine is usually only stocked in limited amounts.  Atropine is also under-stocked in the hospital if you have to manage a cholinergic poisoning.  Elise comment: In a pinch, you can have a patient with a cholinergic toxidrome drink atropine eye drops which actually has a large amount of bioavailable atropine.

 

46yo male took 35 tabs of Verapamil SR.  Atropine, Ca, glucgon, insulin/glucose all given with no significant improvement.  The poison control center found that the emergency physician was under-dosing all the above meds.  Axiom:It’s toxicology, not homeopathy.  Give the patient enough antidote!  Ingestions that need a lot of antidote: CCB’s, Beta blockers, organophosphates, botanical glycosides, flourides, clonidine, sodium channel blockers, INH, gyromitra.

 

Patient ingested OCP’s and Etoh in a suicide attempt. Pt was deeply comatose. ETOH level was 110.  Initial APAP/ASA, ABG is normal.  Repeat labs showed anion gap and metabolic acidosis.  Toxic alcohols were drawn late and were negative.  Pt still was dialyzed and found to have toxic alcohol ingestion.  Toxic alcohols were low because they had been metabolized by the time the blood was drawn.  Toxic alcohols are more intoxicating gram per gram than ETOH.  Axiom: There are toxicologic hidden killers.

 

Hidden or delayed killers: APAP, Iron, MAO’s, methanol, amanita mushrooms, sustained release drugs, and illicit drug packets.

 

22yo female possibly took whole bottle of theophylline.  Charcoal given.  Theophylline level was therapeutic.  Pt admitted to psych.  9 hours later pt had seizures due to a theophylline level that was  toxic.  Axiom: Sometimes you need to get serial drug levels.  Serial levels advised for asa valproate (SR), lithium (SR), carbamazepine (SR), CO from methylene choloride, and theophylline.

 

16 yo male with a seizure at a party.  Axiom: Most tox patients will do well with supportive care alone.  The top 2 things show to improve mortality are managing the airway and controlling hyperthermia.   Other important management items are treating hypotension, correct acid/base disorders, control seizures/agitation.

 

Negro     Safety Lecture     The Agitated Patient

 50% of physical attacks that occur on healthcare workers happen in the ED.

Our main job is to gain control of the situation, protect staff, and rule out an organic cause for the agitation.

These cases have high physician medico-legal risk and have a high patient morbidity and mortality.

Agitation can be primary or secondary.  Primary causes are due to patient’s psychological response to stressors such as police, family, financial stress.  Secondary causes are drugs, psychiatric illness, intracranial hemorrhage, sepsis, etc.

Elise comment: Never let an agitated patient get between you and the door.  Residents’ comment: Be aware of your safety in the quiet room.  Do not let too many people into the quiet room when you have to give the family bad news.  Make sure you can get out the door at all times.

Indications for restraint: prevent harm to patient, staff, or other patients.

Physical restraint: This is a team activity.  You should have 5 staff members to physically restrain a patient.  Never place the patient in prone position. The patient should be supine.  Arms should be restrained one up and one down.  Left leg restraint should be tied to right of cart and right leg restraint should be tied to left side of cart.

 

*patient physical restraint.  Only quibble with this image is that leg restraint should be tied to opposite side of cart so patient can't kick out.

 

Chemical restraint: Using a drug to manage a patient’s behavior or limit their movement.

Start with a benzo.  Long acting benzo (Ativan) usually preferred.  Second drug is an atypical antipsychotic such as IM Geodon.   Pharmacist comment: Geodon can prolong QT interval.  Elise comment: If I have a suspicion of a risk of long QT, I would stick with higher dose benzos +/_ ketamine 5mg/kg IM or 2 mg/kg IV instead of using antipsychotics.

Typical psychotics that can be used are droperidol or Haldol.  These also carry risks of QT prolongation.  

Febbo comment: don’t mix zyprexa with benzo’s due to risk of increased respiratory depression.

Weingart suggestion (EMCrit): 5mg droperidol and 5 mg of versed IM.

Agitated or Excited Delerium: pt is completely out of control,  pt is psychotic, vitals show tachycardia, tachypnea, htn.  Pt has incredible strength.  Many deaths have been reported.  Taser-related deaths are commonly associated with this disorder. These patients require rapid chemical restraint.  They are very dangerous to themselves and others.

Iannitelli comment: Be aware of personal risk when you approach a patient to examine them.  Place your hand on the patient’s closest arm so you can push away rapidly if they reach for you.

 

EM-3 Farewell Lecture

 

Conference Notes 6-4-2014

 

Purnell/Patel             Oral Boards

Case 1.    15 month old presents with  choking/drooling/possible seizure.   Temp is 37.7.  No PMH and no meds.  By the time the physician examines the patient the event has resolved.   CXR shows coin lodged in esophagus at clavicles (thoracic inlet).          Critical Actions:  Consult GI for endoscopic removal of coin.    Most coins (70%) get stuck at the thoracic inlet.  15% lodge in mid esophagus and 15% at lower esophageal sphincter.   

 

Case 2.   33 yo male presents with no pulse.  Pt was electrocuted while working on power lines.  Rhythm on monitor is ventricular fibrillation.  Patient requires defibrillation.    Labs show markedly elevated CK.  Patient also has a compartment syndrome of the forearm.   Critical Actions: Defibrillate,  start amiodarone, give aggressive IV hydration for rhabdomyolysis,  get emergent surgery consult for fasciotomy for compartment syndrome.   Update Tdap.   Cosnult Burn Center.    Elise comment: Don’t say I want to shock the patient.  You need to specifically say either “I am going to cardiovert the patient” or  “I am going to defibrillate the patient”.  Any time patient’s clinical status changes, be sure to re-evaluate the whole patient just as you would do in real practice.   Be sure to differentiate between escharotomy and fasciotomy.   Escharotomy is cutting through severely, circumferentially burned skin to relieve tissue compression.  This patient had compartment syndrome for electrical burn and needed fasciotomy.  Most likely mechanism of injury is heating of bone which has high electrical resistance and  resultant injury to surrounding muscle compartments.  Also, in the patient who suffered severe electrical injury consider other injuries such as C-Spine injury or head injury.

 

*Fasciotomy (google image)

 

*Escharotomy (google image)

 

Case 3.     27yo female presents with generalized seizure.  Patient also has fever/tachycardia/hypertension.  Urine pregnancy test is negative. Patient was travelling through airport and developed seizure while in customs.      Critical Actions: Initial Ativan terminates seizure.   Patient was intubated for airway protection when her mental status did not clear and she had pooling secretions in her airway.  Patient was found to be a drug packer.  IV Ativan given again.  IV phentolamine was also given for severe hypertension.  Consult surgery for emergent laparotomy.   Difference between packer and stuffer: Packers carefully wrap drugs to prevent spillage and put a large amount of these packages in their gut.  Stuffers rapidly put un-packed drug in their mouth and swallow it to avoid arrest for drug possession by unexpected arrival of  police.

 

*Drug Packer (google image)

 

*Drug packages removed from above patient (google image)

Maletich comment: If you are going to lap a patient should you be giving whole bowel irrigation?  Elise comment: If patients are toxic from the drug they packed, you go to surgical mode and patient should get an emergent laparotomy.  WBI is only for asymptomatic packers.  Christine comment: Be aggressive with benzos in the management of cocaine toxicity.  Elise comment:  Any oral boards patients from the airport will likely be a tox case or  an infectious disease case.

Harwood comments:  patient 1 needs to get endoscopy,  patient 2 needs to go to a Burn Center,  patient 3 needs to go to OR.  The choice of phentolamine for hypertension related to cocaine toxicity was excellent.

 

EM/PEDS      Joint Conference    Appendicitis

There still is a lot of controversy  regarding the best management of appendicitis.

In a patient with high likelihood of appendicitis do you need imaging?  Lobe response:  If the diagnosis is clear cut I don’t need imaging.  Other surgeons present felt ultrasound as the first imaging study was safe and indicated in most patients being evaluated for appendicitis. 

Mary Ann Collins comment:  In uncomplicated appendicitis we chose cefoxitin antibiotic coverage peri-operatively based on antibiograms for gram negative gut flora and anaerobes.

Lam comment: Approximately 50% of ultrasounds for pediatric appendicitis are non- diagnostic.  Collins and Slidell comments:  Radiology is working to decrease their rate of non-diagnostic ultrasounds.   CT imaging does carry radiation risk.   You can lower the dosage of radiation for CT’s in kids.  This is protocolized for all kids getting an appendicitis CT study here at ACMC.  

Lo comment: A non diagnostic U/S is not entirely unhelpful info.   The fact that nothing was seen on the U/S does lower the risk of appendicitis.

Slidell comment: MRI for pediatric appendicitis works well.  It may be something we will start doing in the near future.   2013 study of MRI for appendicitis showed 100% sensitivity and 96% specificity of MRI for appendicitis with no radiation exposure.   This modality is probably useful only for teenagers due to the need for the patient to be very still.

In a patient with moderate probability for appendicitis: When the ultrasound is indeterminate,  is there a role for serial exams instead of going to CT?    Panel all agreed that serial exams have a role.

If U/S is read as negative, does that rule out appendicitis?  Panel agreed yes it does.

If U/S is positive for appendicitis can the patient be treated with antibiotics instead of surgery?   Surgeons’ comments: There is a limited role for antibiotics as definitive appendicitis management but we don’t have good criteria to select which patients will succeed with antibiotic management.

There was agreement that if there is a radiologic diagnosis of appendicitis and OR is planned for several hours later then starting IV antibiotics is indicated. 

Harwood comment: Which antibiotic and when is the best option for uncomplicated appendicitis that is waiting to go to OR?     Slidell and Collins response is the answer is not clear.  Very muddy waters.  Cefoxitin is a reasonable choice.  Surgeons prefer that ED physicians start antibiotics in ED.

 

Slidell      Un-Complicated Appendicitis

There is much variability both nationally and internationally regarding the management of appendicitis.

Atypical presentations occur in 50% of cases.  The diagnosis is more difficult in kids less than 4 years old, females, and those with co-morbid illness.

There may be different types of appendicitis that have different courses of illness.

Most hospitals have a negative appendectomy rate from 0-17%  median is about 2.5%.

Median range of ruptured appendicitis is 37%.

Median LOS for uncomplicated appendicitis is 1.8 days.  Ruptured appendicitis patients stay for 5.2 days.

Laparoscopic surgery is the predominant approach for surgical management of appendicitis.

Risk factors for complicated appendicitis: poor healthcare access, children under 4 years old, co-morbid illness.

No increase in perforation rate or morbidity if surgery is delayed 12-24 hours.  In general outcomes are better during day time compared to night time surgery.

Timing of surgery within 12-24 hours of presentation does not affect outcome or complication rate of appendicitis.  There are studies showing that delaying surgery beyond 24 hours does increase perforation risk.   Bill Schroeder comment: What is the start time for all these studies, onset of symptoms, time of presentation to ED,  or time of radiologic diagnosis?  Slidell response: It is not standardized in these studies.  My take is that we should operate as soon as possible and 12-24 hours from ED presentation seems reasonable.

Children operated in the morning after overnight antibiotics had lower rate of abscess formation.

Antibiotics for appendicitis: Peri-operative antibiotics for 24 hours in un-complicated appendicitis.    Perforated appendicitis gets about a week of antibiotics.   Gangrenous appendicitis falls somewhere in-between.  At ACMC we include gangrenous appendicitis in the perforated group and manage them with 7 days of antibiotics.

Treatment of appendicitis with antibiotics only has a success rate of  about 70% vs. 97% success rate of surgery. (Cochrane review of mostly adults).  20-50% of patients treated with antibiotics alone will eventually need surgery for appendicitis within the next 12 months.

Small study of kids recently showed that 27 of 30 kids managed with antibiotics only had no problems by 6 moths. 

Possible risks factors for failure of antibiotic  therapy are: fecolith on imaging, duration of symptoms more than 24 hours, WBC>18, elevated CRP.  There is a current multi-center study based at U of C  assessing this issue.

Harwood comment: Kids managed with antibiotics will need imaging their whole life if they have abdominal pain.  Also it has been shown that patients treated with antibiotics tend to have a higher incidence of chronic abdominal pain.

Some surgeons make the analogy that non-complicated appendicitis is essentially a version of diverticulitis.   If you treat with antibiotics, it makes the surgery no longer an emergency.  Appendectomy becomes semi-selective once antibiotic therapy has been initiated.   This is not the concept that lay people have known previously.  Harwood comment: The analogy is not perfect.  You have one appendix and potentially hundreds of diverticuli.   Diverticulitis has a high cure rate with antibiotics @99%.  Appendicitis cure rates with antibiotics are @70%.  There is a very low risk surgical procedure to treat appendicitis.  Surgery for diverticulitis has higher risks and at times results in colostomy.  So uncomplicated appendicitis has worse cure rates with antibiotics and lower risks associated with surgery than diverticulitis.

 

Advanced Procedure Lab

 

 

 

 

 

 

 

Conference Notes 5-28-2014

 

Chan        Study Guide   Genitourinary

 In a female with recurrent uti’s you have to treat presumptively  for STD’s.   Treat with ceftriaxone and azithromycin and flagyl.    Elise and Harwood comment:  You should treat for GC/Chlamydia/Trich.    Also you should consider interstitial cystitis as a diagnosis.   Girzadas asked the question if other faculty are checking EKG’s for long QT interval prior to prescribing Zithromax.  Most faculty felt Girzadas was being overly compulsive.  Girzadas said he has picked up a couple of patients with QT intervals over 500 with  screening ekg’s.   Most faculty were not swayed. 

 Fournier’s gangrene:  polymicrobial etiology,  aggressive infection in genital/ perineal region.   Requires emergent surgical debridement.   Most commonly this occurs in poorly-controlled diabetic males.  Antibiotic coverage should include gram positives & negatives and anaerobes. 

There was a discussion among faculty about asymptomatic bacteriuria in pregnant patients.   Elise and Kelly get a formal urinalysis on pregnant patients in the ED to look for bacteriuria.  If they find it, they treat for a week.   Harwood does not look for bacteriuria with a formal urinalysis.  He just does a urine dip and makes a treatment decision based on that.   Resident reports about our OB clinics is that they don’t routinely screen for bacteriuria.  Faculty felt they have a different patient population than we do and we may need to look for this more aggressively.

Up to Date Reference: Bacteriuria has been associated with an increased risk of preterm birth, low birth weight, and perinatal mortality. In clinical practice,  only one screening voided urine specimen is typically obtained, and treatment is usually started in women with ≥105 cfu/mL without obtaining a confirmatory repeat culture. Multiple studies have been performed to assess whether rapid screening tests, such as dipstick, enzymatic screen, reagent strip, or interleukin-8, might have comparable sensitivity, specificity, and predictive value to urine culture for the detection of asymptomatic bacteriuria in pregnant women [20-22]. These methods do not come close to urine culture in terms of sensitivity and specificity and should not be used.

Girzadas comment: It looks to me like you probably need a urine culture to definitively diagnose asymptomatic bacteriuria.

 There was a discussion about the difference between the terms pathognomonic and sine qua non:   Pathognomonic is a very specific symptom or sign.   Sine qua non is a very sensitive symptom or sign. 

 Treatment of priapism: Aspirate corpora cavernosa.   If that doesn’t solve the problem,  and inject 1ml of diluted phenylephrine into the corpus cavernosum every 3-5 minutes until resolution.   Harwood comment: you don’t have to inject both sides.  There is a vascular communication between both corpora cavernosum.

Attempt at detorsion of testicular torsion should be the technique of “opening the book”    If you get partial relief with 180 degrees of detorsion it is ok to detorse further.  If your attempt worsens pain,  try using the “close the book” technique.  Pt’s still need surgery after successful manual detorsion because they are at risk for recurrent torsion.

 

 

*Open Book Detorsion Technique

 

Epididymitis: >35 yrs old is more likely due to ecoli.   <35 is more likely STD  #1 is chlamydia and #2 is GC.  If you get an U/S diagnosis of epidydimitis in a child, Cindy said she was told by Dr. Nold that the diagnosis is more likely torsed appendix testes.  Most faculty would manage this child with a urine culture, starting antibiotics to cover ecoli and give an NSAID for possible torsed appendix testes.  Kelly comment: There is literature supporting conservative management with no antibiotics in kids with an ultrasound diagnosis of epididymitis.  Treat only if pyuria is present or urine culture comes back positive.

 

3 anatomic locations where kidney stones get stuck in the ureter: ureteropelvic junction, pelvic brim/crossing over iliac vessels, ureterovesicular junction.   If a stone makes it to the disal third of the ureter, 70% will pass spontaneously.   

 

*Kidney stone anatomic areas of getting stuck

 

Balanitis= inflammation of the glans.   Balanoposthitis=inflammation of glans & foreskin.

 Paraphimosis reduction techniques include: compression with ace bandage,  osmotic agents, puncture technique, forceps retraction, and dorsal slit procedure.

 

Paraphimosis puntcure technique 

 

Avoid urinary catheterization in patients who can void spontaneously and can sit on a bed pan/commode or ambulate to bathroom.  You want to avoid catheter associated uti’s in these patients.

 

Intraperitoneal bladder rupture: gross hematuria in 98% of patients.  Usually it occurs in a patient with a full bladder who suffers a compressive force (MVC) causing bladder to rupture at dome.   Extraperitoneal bladder rupture usually is due to a pelvic bone fragment perforating the bladder. 

*Intraperitoneal bladder rupture (you can visualize bowel loops due to contrast)

 

*Extraperitoneal bladder rupture

 

Negro/Herron        Oral Boards

 

Case1.   50yo male exposed to unknown gas in a train station.  Multiple casualties at this incident.  Pt has diarrhea and vomiting.   HR=40.   SLUDGE (cholinergic) toxidrome.  Diagnosis: Nerve gas (organophosphate) poisoning.    Critical actions:  decontamination,  personal protective equipment Atropine hi dose,   2-PAM (Pralidoxime), , airway control if needed.    Avoid succinylcholine in these patients because there is competitive inhibition of the succinylcholine by the cholinergic agent.  Use rocuronium in higher than normal dose    Kelly , Harwood, and Elise comment: Step one in managing these patients is using personal protective equipment, decontamination of the patient, getting unnecessary personnel away from the resuscitation room.   EMS personnel know this but ED docs don’t do this naturally.  We need to address resuscitation safety for our team first before we start treating the patient.   Andrea comment: Your first call should be to a poison center.  They can mobilize atropine resources.  You frequently will use up your hospital resources of atropine.   Girzadas comment:  In a previous conference notes it was discussed that having a patient drink atropine eye drops actually can provide a very large dose of atropine for a patient in a mass casualty event if IV atropine is not available.

 

Case2.     23yo female with  lower abdominal pain and shuffling gait.   On exam, patient has CMT.   U/S is negative.  Diagnosis: PID with peritonitis.    Critical actions: IV antibiotics, admission.   Not all PID patients need to be admitted. Criteria for admission for PID : pregnancy, non-compliance with treatment,  not improving with oral meds, TOA, high fever, nausea and vomiting, severe abdominal pain/peritonitis, need for surgery, other possible diagnoses like appendicitis.

 

Case3.     15yo male presents pulseless after sustaining blunt trauma to chest in hockey game.   EKG shows V-fib.      Diagnosis: Commotio cordis    Critical actions:  Defibrillate,  IV Epi, CPR,   Intubate.  Therapuetic hypothermia. Elise comment: Don’t intubate this patient first.  The key thing is defibrillation.  Shock first and then do all the other stuff like intubation and meds second.

 

*Commotio cordis

 

Maletich    M & M

 

24 yo female with a CC of flank pain.  Patient was diagnosed with a kidney infection and started treatment with macrobid  the day prior.  The clinical setting in the ED was very busy, rife with distractions.    At this ED visit, the patient had bilateral flank tenderness and abdominal tenderness on exam.  Jim orders a CT of abdomen and pelvis with IV contrast.  Labs significant for HGB =8.9.  No UCG available yet.  The ED attending intervenes at this point and performs bedside U/S.  U/S shows fluid in Morrison’s pouch.  The attending asks the patient are you pregnant?  Patient states yes.   The management  of the case rapidly changes gears to resuscitation of a hypotensive ectopic pregnancy.  In OR patient  was found  to have a ruptured cornual pregnancy.

 

Jim questioned how this occurred.  Why was the patient bradycardic, and why was there no vaginal bleeding?

There are multiple papers describing that vital signs fail to correlate with hemoperitoneum in ruptured ectopic pregnancy.  This postulated to be due to increased parasympathetic tone from peritoneal stimulation.

 

*ectopic pregnancy location

 

Cornual ectopics are also known as interstitial ectopic pregnancies.  These are in the muscle where the fallopian tube originates.  These occur later in pregnancy.  When these rupture, they can bleed quite a bit.  Cornual pregnancies account for 2% of ectopics but unfortunately also 44% of the deaths.   Harwood comment: The artery in the cornual section of the uterus is large and if there is rupture  in that vicinity they bleed severely.

 Blood in the wrong place is painful.  Blood irritates the peritoneal cavity.  

Sola comment: The pregnancy test in a child-bearing age female is really equivalent to a vital sign.

You can  use the patient’s blood to run a urine pregnancy test if you are facing delays getting urine.   

Anna Sklar’s comment: This case is an example of framing bias.  The patient was triaged to a lower acuity area of the ED.

Andrea’s comments:  We are all pattern seekers and building your pattern on the report of an inexperienced student’s exam is fraught with hazard. You really need to see the patient yourself so that you don’t have premature closure based on an incomplete or inaccurate pattern.

Sola comment: If there is a long wait for a CT scan, get an U/S to the bedside.

Elise comment: Our own emotional baggage taints how we view the patient.

Girzadas comment: These are all great comments describing how different biases affect our thinking when we are at work.  Often when most needed our thinking is foggy when we are working in a busy ED environment. When we are working in the ED our thinking is not as clear as when we are sitting in the conference room discussing the case.

McKean comment: This patient is in shock and is not making much urine.  That is why it was tough to get urine.

Harwood comment:  Bile and gastric fluid are the most irritating fluids to the peritoneum.   You will see relative bradycardia due to blood in the belly multiple times the rest of your career.  It is not that uncommon.   You can send blood for a qualitative HCG in the lab which is pretty quick  or you can run the urine preg test with the patient’s blood.

Anna Sklar comment: Give the patient the benefit of the doubt.  Before you chalk up the patient’s behavior to dramatics give them all your thought and effort to rule out life threats.

 

Cash/Lovell      Resident as Clinical Teacher

 Elise showed a funny video of a pedantic surgeon teaching young surgeons at the bedside.  Pedantic teaching techniques don’t work very well: Aggressive yelling, not being respectful of learners, focusing on minutia, not giving learners time to think, ignoring the patient. 

 Characteristics of Great teachers:  Energy level, approachable, enthusiasm, engaging.  They take the time to ask you challenging questions.  Hold you accountable.  Give you usable feedback.    Elise comment: It is optimal to highlight feedback to the learner by saying explicitly, “I am going to give you some feedback now.”   

The literature says great teachers have very positive non-cognitive skills like positive relationships, communication, and enthusiasm.    Your medical knowledge and technical skills are important but are considered a baseline that all teachers must have.   The non-cognitive skills separate the adequate teachers from the great teachers.  

Excellent teachers state these are important behaviors: tailor your teaching to the learner.  Actively involve the learner.  Give the learner responsibility for their learning.  Actively seek opportunities to teach.   Agree on expectations.  Be a role model. Thinking out loud is a good way  to convey your thought process.

 Traditional teaching: Teacher-centered method .  Teacher functions as expert consultant. Learners are passive.  Learners don’t express clinical reasoning skills.

Newer teaching model: Learner-centered teaching.  Teachers focus time on finding knowledge gaps and teaching to that gap. Teachers act as facilitators.  Learners ask more questions and actively pursue knowledge.

One minute preceptor model:

  1. Get a commitment from the learner.  Have them say what they would do in the situation.
  2. Probe for supporting evidence.  Identify the learner’s knowledge gaps.
  3. Teach a principle.  Give a pearl to the learner.
  4. Reinforce what was done well.  Give positive feedback on what they knew or what they did well.
  5. Correct mistakes
  6. Identify next learning steps.  Encourage the learner to continue independent learning after the encounter.

These don’t all have to be done every time.

The residents then practiced using this model.

Unfortunately I missed the rest of this outstanding lecture due to a meeting. 

Conference Notes 5-21-2014

 

Ireland     Patient Satisfaction

 People don’t cut you slack when comparing  service.  They know if it is good or bad.  They will compare their experience in the ED with their experience at Disney or ordering from Zappos.

Who gets to decide how the experience was?   Not the doc or the nurse but the patient.   

Customer service is a moving target. Each year the bar gets moved higher.

75% of your workday involves work.   Why not make it a positive experience.   Work on things you can change: interactions with patients, co-workers, your attitude.

Everything is driven by culture.   You have to develop a great culture to be great.   You can design culture to be great.

Very effective organizations like Walmart and Walgreens are now our competition in the healthcare market. If we can’t get service right, we will be “eaten alive” by these big corporations. Culture is “the way we do things around here”  Culture optimally is designed and intentional to get the results you want.  As an example of culture by design, Dr. Ireland’s group story-boarded out what the perfect patient experience would be.

Great service is delivered by  a conscious design of people, place, and process.

Patients can’t assess technical quality of health care.  All they can assess is our service and if they like us.

Quality service=Exceeding  expectations by paying attention to detail, layers of detail.  Treat patients like they are quests in your home.

Doctors miss 90% of empathy cues when dealing with patients.

The main commodity of the 21st century is time.  We gotta understand that to be successful.  People value their time most of all.

Saying thank you to people is critical.  Saying “Thank you” is the #1 way staff want to get positive feedback.

We judge ourselves based on our intentions but we judge others by their actions.  We gotta stop doing that. We gotta judge ourselves by our actions.

68% of customers leave a company or patients leave a practice based on a negative or indifferent employee attitude.  It is the #1 reason people seek another provider.

There needs to be 40 positive customer experiences to counter-balance a single negative customer experience.

What do customers AND employees want:  Treat me special,  treat me as an individual,  treat me with respect, educate and develop me.

ARE=Appreciate, respect, encourage employees and customers.

Everything speaks to the customer.  If a customer goes to a great restaurant  and the bathroom is disgusting, that affects strongly the customer’s assessment of your restaurant.

Great systems deliver great service.  Foster continual improvement, engage everyone, make it hassle free,  Ask patients, ask patients, ask patients what would iprove their experience. Ask staff, Ask staff, Ask staff what would improve their experience. Guide patients through the experience.

There are many benefits of patient satisfaction: increased patient compliance, decreased malpractice risk,  more fun, increased revenue.   Docs gotta be careful because we will have yelp and healthgrade reviews of us on the internet.

Service Profit Chain:The most important part of the chain is internal service quality (how employees treat eachother)  Internal service quality leads to employee satisfaction and employee retention and employee productivity.   These all lead to external service quality which leads to customer satisfaction, customer loyalty and finally revenue growth.

 

 

*Service Profit Chain (google image)

We have medical knowledge that we learned in med school and residency.  But the patient’s perspective depends on how we deliver this knowledge.   We don’t get much training on how we deliver the knowledge we have.

Some good reads to follow up this lecture:  “How to make people like you in 90 secs or less. “    “Hug your customers.”      “What got you here, won’t get you there”     “Creating Magic”

 

Tekwani    Study Guide GI

 Spontaneous bacterial peritonitis= Peritoneal fluid cell count >1000 or PMN’s>250.   Treat with IV  Cefotaxime or in penicillin allergic patients you can use levofloxacin.  Common pathogens are e coli, strep, and klebsiella.

Acute Alcoholic Hepatitis:  AST/ALT ratio >2, hepatomegaly, possibly low grade fever.

The most common cause of acute liver failure: acetaminophen.  Hepatitis B is the most common viral cause of liver failure.

Anal fissure is almost always located in posterior midline.  If fissure is in another location you have to consider crohn’s, syphilis, TB, HIV, cancer, or FB insertion.

Most common complication of cirrhosis: ascites. 

Sigmoid volvulus is more common than cecal volvulus.  Sigmoid volvulus is seen in elderly bedridden patients.  It is reduced endoscopically.  Cecal volvulus is seen in marathon runners.   Cecal volvulus is treated with surgery.  Cecal volvulus on xray shows dilated bowel  that extends up into left upper quadrant and sigmoid volvulus shows dilated bowel to right upper quadrant.   Girzadas comment: I think about these volvulus’ as twisting to the opposite side of the abdomen with the cecum going to the left and the sigmoid going to the right when they develop volvulus.

 

*Cecal Volvulus (google image)

 

*Sigmoid volvulus (google image)

 

Toxic megacolon is treated with IV fluids, antibiotics, and corticosteroids.  Toxic megacolon is more common in Ulcerative Colitis than Crohn’s.

 Bowel rest for inflammatory bowel disease has not been shown to be useful.

The most common complication of diverticulosis is diverticulitis.

Pregnant women infected with Hepatitis E are more prone to fulminant liver failure. Hepatitis has nastier potential than hepatitis A with immunocompromised and pregnant patients at higher risk for chronic and fulminant hepatitis. Hepatitis E is the most common cause of hepatitis world-wide.  Hepatitis A does not cause liver failure.

Hepatitis E (google image)

Most common cause of large bowel obstruction is neoplasm.  #2 is diverticulitis.  #3 is volvulus. 

Most common cause of massive lower GI bleeding: diverticulosis

Lipase is more specific than amylase for diagnosing pancreatitis.  Lipase and amylase have similar sensitivities for diagnosing pancreatitis.

Perianal complications are more common in crohn’s disease than in UC.

Erythema nodosum is more common in crohn's disease but can be related to either crohn's or UC.

Charcot’s triad for acute cholangitis: fever RUQ pain, and jaundice

Reynold’s pentad for acute cholangitis: fever, RUQ pain, jaundice, altered mental status, and hypotension.  Most appropriate management of acut cholangitis is ERCP. IV fuids and broad spectrum antibiotics.

Undercooked shellfish has potential to transmit Hepatitis A.

Best treatment for uncomplicated diverticulitis: oral antibiotics

Complicated diverticulitis: phlegmon, abscess, stricture, obstruction, fistula, or perforation.

In acetaminophen overdose the normal liver pathways of glucuronidation and sulfation are overloaded and acetaminophen is metabolized by the P450 system to toxic NAPQI

 

*Metabolism of Acetaminophen Overdose

 

If you see HBeAG that means active infection of Hepatitis B.

 

 

*Hepatitis B serology

 Girzadas comment: There can be confusion between HBeAG which is the active infection of HepB and Hepatitis E which is transmitted by the fecal oral route similar to Hep A.  

The delta virus can only exist with Hepatitis B.

If patient has conjugated hyperbilirubinemia there is a post-liver obstructive process like sclerosing cholangitis.

Most common cause of anal fistula: intersphincteric abscess.  This abscess is due to  an infection of an obstructed anal gland.

CT scanning in first trimester of pregnancy doubles the infant’s childhood risk of cancer.  However, in absolute numbers, the risk is still low.

Acute radiation protocolitis: diarrhea, rectal pain, tenesmus, abdominal pain.  Treat with analgesics and sucralfate.

 

Katiyar      Am I Really Too Slow?

 RVU compensation was developed in the 1980’s.  RVU is based on time, skill, training, and intensity.   RVU=work produced.   An RVU is assigned to every CPT code based on work, practice expense, and professional liability.  RVU values increase proportionally with increasing EM code.   Levels 4,5,critical care cases have the highest reimbursement.  A level 4 chart brings in a $114.  A level 5 chart brings in $168.   A critical care chart brings in $217.

 There are two common reimbursement models that utilize RVU’s.

Model 1. Eat what you kill: 100% RVU reimbursement to the physician.

Model 2. Piece of the Pie: Guaranteed base salary and a portion of your reimbursement based on RVU generation.

You can maximize your RVU’s by seeing patients as quickly as possible and charting promptly and accurately.

Charting: Chintan’s mnemonic is a made up word “FourTwoTenEight”  or “Fortutenate”.   This word helps you remember  Four HPI elements.  Two PMH elements (PMH/surgical history plus either Social or Family history), Ten ROS items, and eight PE items that will get you to a Level 5 chart.

Abhi gave examples of ER docs being fired for having low RVU productivity.

If you don’t document your work well, it severely affects your RVU production for a shift.

Top 5 Critical care misses (not billing for critical care when it is appropriate): agitated patients, tertiary care transfers, overdose management with antidote, hypoxia with O2 sat<87% and treatment of hyperkalemia with IV drugs.   All STEMI’s are critical care.  If the patient does not stay in the ED for the full 30 minutes you can still legitimately document critical care if you chart after the patient leaves the ED or you discuss with family or consultants after the patient leaves the ED. 

Ways to increase RVU’s: document your interpretation and management of cardiac monitoring.  Document pulse ox/peak flow.  Document your procedures.   Always measure the length of lacerations.  Document layers, location, and if any foreign body or debris. 

To measure laceration lengths,  the paper wrapper of the cotton swab and 4X4 gauze have a ruler printed on them.

Packing abscesses increases the RVU for abscess management.

Documenting  that you checked the neuro-vasc status of the patient after the tech has placed a splint increases your RVU.   Sam comment: If you place your own splint you also increase your RVU’s.

Remember you are responsible for the accuracy of the bill no matter who does your billing.

 

Knight   Safety Lecture

Surgery was consulted to repair a dog bite wound of the patient's ear. Patient was sedated prior to arrival of consultant.  There was a patient handoff in the middle o this case.  Consultant then notified ED that they were not going to be able to perform the procedure.  We then had a patient sedated without a consultant to manage the problem. 

 We don’t have 24/7/365 coverage for peds plastics, peds neurology, peds facial fractures.

In the end, the ED physician repaired the wound in the ED and arranged plastics follow up for patient with plastic surgery.

Safety : Assure clear communication between hand off teams and consultants.  Verify arrival times of consultants.  Avoid sedating patient prior to arrival of consultant.  Document all interactions with consultants.

 Andrej and Elise comment: It is important to clearly convey the severity of wounds to consultants.  A texted image of the wound is probably the best way. 

Elise comment: Don’t sedate a patient prior to verifying that the consultant is in the ED to manage the problem.   Start working on transferring the patient early in the ED course if you don’t have the required specialist on call.  Harwood comment: It is important to communicate well with the family.  Set up the situation that you are willing to repair the injury but that you are attempting to speak with the plastics specialist. Also inform the family that your child may need to be transferred.    

Conference Notes 5-14-2014

 

Balogun/Collins    Oral Boards

 

Case 1.   30yo male, s/p mvc  with head injury.  Transient LOC.  Patient has headache On further history patient is found to have hemophilia.  CT shows ICH.   Critical Actions: Give Factor 8 prior to CT.  You want to get to 100% activity by giving 50u/kg of Factor 8.   Get stat CT head.  Consult hematologist & neurosurgeon. 

 

Case 2.   30yo female with 1 day of left shoulder and flank pain. Patient had syncopal episode yesterday and fell down some stairs.   Patient is hypotensive.   Pregnancy test is negative.   FAST scan shows fluid in Morrison’s pouch.  Patient had splenic rupture. Critical actions: Resuscitate with IV fluids and PRBC transfusion.  Consult surgery.  Admit to ICU.  The spleen is the most common organ injured in blunt abdominal trauma.

 

*Fluid in Morrison’s pouch (google image)

 

Case 3.  30yo male firefighter is SOB and confused after being pulled from building fire.  Patient has burns on trunk and face, singed nasal hairs.   Critical Actions:  Intubate based on likely airway burns.   Administer parkland formula IV LR.   Patient had elevated CO level and needed hyperbaric therapy.  Indications for hyperbaric therapy: syncope, altered mental status, seizure, pregnant with level >15%, blood level>25%, or myocardial ischemia.

 

*Parkland Formula (google image)

For pediatric patients less than 20kg, add maintenance fluids to the calculated Parkland fluid dose.

 

Christine comment: It is critical to give Factor 8 prior to sending hemophiliac patients with head injury  to CT.

Girzadas comment: Consider inter-personal violence as an underlying cause of women and children presenting with trauma.   Know the Parkland formula for adults and remember in kids less than 20kg you have to add maintenance fluids to the calculated Parkland Fluid volume.

 

Herrmann/Ketaneh     STEMI Conference

 

Case 1.  67yo female presents with flank pain.  She has a history of CABG.  First EKG does not have STEMI criteria.  Patient developed chest pain in ED.  2nd EKG shows LBBB with lateral ST depression.   Silverman comment: Patient is tachycardic on second EKG and LBBB could be rate-related.   I would slow her rate with a beta blocker and see if EKG improves.  Labs show elevated Cr and potassium= 6.4.  Troponin is 0.16.   Interventional cardiology did not take patient to cath lab initially.  They preferred to treat potassium and treat pain.  3rd EKG shows tachycardia and LBBB.   Cardiologists in audience comment: They generally felt hyperkalemia is the issue and the risk of catheterization in the setting of hyperkalemia and elevated creatinine is dangerous.   A STAT echo looking at the anterior wall motion would be helpful in determining if the LBBB was due to STEMI.   4th EKG shows resolution of LBBB with peaked anterior t waves and lateral ST depression.   Patient still has chest pain.    Code STEMI called by the ED physician and patient had a thrombectomy of saphenous vein graft and stenting of another vessel.   Cardiologists in the audience did not feel that a Code STEMI should have been called.  Cardiologist comment: To call a STEMI in the setting of a LBBB, the patient needs to have ischemic chest pain and have a clinical picture of ACS.   Most would not have taken patient to cath lab with hyperkalemia and elevated creatinine. 

 

Case 2. 69yo female presents in respiratory distress.  She was initially treated for COPD exacerbation.   Initial EKG shows non-specific changes and is made more difficult to interpret  due to a lot of baseline artifact.   Patient had respiratory acidosis on ABG.  Patient was intubated.  Repeat EKG shows tachycardia, lateral Q waves and ST elevation.   Cardiologists in audience felt the 2nd EKG did not make sense with the clinical picture or in comparison to first EKG.  They questioned whether lead placement was different.   Q waves don’t usually develop in 2 hours.   Cardiologists in the audience looked at the first EKG again and felt there was subtle ST elevation in V6.   Takasubo’s Cardiomyopathy can present in this fashion.  

 

Takotsubo cardiomyopathy, also known as transient apical ballooning syndrome,[1]  is a type of non-ischaemic cardiomyopathy in which there is a sudden temporary weakening of the myocardium. Because this weakening can be triggered by emotional stress, such as the death of a loved one, a break-up, or constant anxiety, it is also known as broken heart syndrome.[3]Stress cardiomyopathy is a well-recognized cause of acute heart failure, lethal ventricular arrhythmias, and ventricular rupture

The typical presentation of takotsubo cardiomyopathy is a sudden onset of congestive heart failure associated with ECG changes mimicking a myocardial infarction of the anterior wall. During the course of evaluation of the patient, a bulging out of the left ventricular apex with a hypercontractile base of the left ventricle is often noted. It is the hallmark bulging out of the apex of the heart with preserved function of the base that earned the syndrome its name "tako tsubo", or octopus pot in Japan, where it was first described.[5]

Evaluation of individuals with takotsubo cardiomyopathy typically includes a coronary angiogram, which will not reveal any significant blockages that would cause the left ventricular dysfunction. Provided that the individual survives their initial presentation, the left ventricular function improves within 2 months. Takotsubo cardiomyopathy is more commonly seen in post-menopausal women.[6] Often there is a history of a recent severe emotional or physical stress.     Wikipedia

 

Takotsubo cardiomyopathy (google image) Notice how the apex does not contract as well and in effect balloons out.    Sorry for wandering off into takotsubo cardiomyopathy when it is really not relevant to this case.

 3rd EKG in ICU shows Inferior/Lat ST elevation and subtle ST depression in 1 and AVL.   Patient wen to cath from ICU and found to have a  90% occlusion of left main (not takotsubo’s).  Balloon pump was placed.  Patient developed multi-system organ failure and died.  Cardiologist comment: Do bedside echo looking for wall motion abnormalities to help you figure out these cases.  Sometimes if the patient is too sick to go to Cath Lab, you need to document your decision making to not go to the Cath lab.   If you need a rapid read on a STAT echo during daytime hours call 41-5555 to notify the Cardiologist of the urgency of the case.

 

Teaching Points: ST elevation in AVR is concerning for ischemia.  If you identify ST elevation in AVR with associated diffuse ST depression in 6 or more leads this identifies left main occlusion and is considered to be a STEMI criteria by the AHA.   ST elevation in AVR is also seen in LAD occlusion, triple vessel disease, and sub endocardial ischemia.  

Prominent T wave in V1,  V1>V6 suggests early ischemia as well.

 

Gore       Trauma  Lecture

 

Kasia gave multiple pointers for pre-resuscitation preparation when you are on the Trauma service and awaiting the arrival of a patient via EMS.   Talk with the ED nurses prior to arrival of the patient to get an idea of how severely injured the patient is.   Think  ABC’s: Prepare the resuscitation room for intubation.  If you have any info from prehospital personnel that the patient may need a chest tube or thoracotomy, etc., prepare for those as well.  For circulation support prepare for venous or bone access.   First choice line is 2 large bore IV’s.  Second choice line is Cordis placement.   Whatever IV access you obtain, shorter/large bore catheters are preferred over longer/smaller bore catheters.  The shorter and larger diameter catheters allow for much higher flow rates based on poiseuille’s law.

 

 

*Poiseuille’s Law  (google image)    Note that Radius (r4) decreases resistance to flow to the 4th power and length increases resistance to flow.

 

Omi comment: I prefer not to use IO lines because I have seen osteomyelitis following IO line placement and large volume infusions thru IO lines is painful.  We manage mostly young adult trauma patients in whom IV access is usually not a problem.

 

If you have concerns about moving the C-Spine during intubation, one option is to leave the patient in the collar and use the glidescope to intubate while the patient is immobilized in the collar.

 

Discussion about standard “go-to” RSI drugs for the trauma patient: etomidate and succinylcholine or rocuronium where the first choices.   Dr. Omi did not feel the risk/time/benefit ratio favored using fentanyl, lidocaine in addition to the basic two drug RSI regimine nor did it favor a sedated look (ketamine/topical anesthetic).  Omi was strongly against propofol due to risk of hypotension.   Shayla comment:  There is virtue in keeping your RSI plan simple.  Many ED’s don’t have multiple drug options available and they are not used to giving drugs other than for example, succinylcholine for neuromuscular blockade.  

 

Chest Tube Placement:  Kasia showed a video of chest tube placement.   One thing I learned from Dr. Omi is that when she anesthesizes the chest wall prior to procedure, she actually enters the pleural space with the needle.   She will advance the needle until she aspirates air or blood.  Then she withdraws the needle until she no longer is getting air or blood and then injects local anesthetic to numb up the pleura.  She states the pleura is the most painful structure when placing the tube.

 

Shayla comment: Be diligent to look for the second GSW.  Patients frequently don’t realize they have been shot more than once.  They are focused on the most painful or visible injury. 

 

Case 1.  36 yo patient in MVC.  Patient was tachycardic and tachypneic with  flail chest with paradoxical movements of left chest wall and subQ crepitence.  

Kelly comment:  If you are in a community ED with limited staff, just intubate this patient and throw in a chest tube right away.

Patient was intubated and left chest tube was placed.   A right side chest tube was also placed based on hypotension and chest wall crepitence.  FAST exam was negative.  Left chest tube drained 1500ml of blood. (Indications for thoracotomy is 1200ml initially or 200ml/hr for 3-5 hours)  CXR showed wide mediastinum.  Patient was taken to OR for thoracotomy.  Transexamic acid was given in the ED.

Patient eventually died due to multiple severe injuries and coagulopathy.

Shayla Garrett and Ellen Omi comment: If you are working in a rural ED, intubate the patient, place a chest tube, start blood, and helicopter the patient to a Level 1 Trauma Center.  

 

Garrett-Hauser    Ethics

 POLST Form    Physician Order for Life Sustaining Treatment.  Intended for persons for whom death within the next year is not unexpected. This form augments but does not replace the power of attorney form.

Case 1.  14 yo female with abdominal pain and vaginal d/c.  Patient doesn’t want the ED doc to speak with her parents.  Patient refuses treatment for PID.   Can she refuse?   The key issue is capacity for decision making.  If the patient has decisional capacity you could make the case to let the patient decide.   Girzadas comment: I would definitely speak with the parent.  This case has a high risk of bouncing back with a bad outcome if you don’t discuss with parents and/or treat patient.  In retrospect, everyone will question your assessment of capacity.  This patient is not an emancipated minor and her ability to understand the long term consequences of her decision is questionable.

 Case 2.  Jail inmate refused blood transfusion.  Patient however had critically low HGB and was judged by ED physician  to not have decisional capacity because of the severe anemia.  Patient was given PRBC’s against his will.   The group veered into  a discussion on physicians’ roles in judicial executions and forced feeding of inmates. The AMA states that physicians cannot ethically take part in executions or forced feedings.

Case 3.  Police had a court order for a doctor to perform a body cavity search for drugs on a patient brought to the ED.   Patient had no medical emergency identified related to drug overdose.   Shayla’s advice was to not perform a body cavity search against the patient’s will if there is no medical emergency.    A local Indiana hospital is being sued for forcibly obtaining a urine sample to get a drug screen and drawing blood for etoh level for police use despite no medical emergency  being present.   In Illinois, if police ask you about test results for a patient who is under arrest you can give that info to police.

 

Case 4.  Patient with ruptured ectopic.  Patient refuses to consent for surgery.  She says this is God’s will that this will kill me.   Patient would only consent to blood transfusions.  The emergency physician activated psychiatry, risk management, administration to help solve the problem.   Psychiatry saw patient and felt she did not have decisional capacity.  So surgery was done.  

 

Case 5.  Patient is on a liver transplant list.  Patient presents to ED following a fall with head injury.   Blood alcohol level is 250.  Should the emergency physician notify the transplant center that the patient’s alcohol level is 250? In most centers this info will knock a patient off the transplant list.   There was a split in the group about what to do.  The majority of the group would report the patient to the transplant center.   The breakdown was basically along concerns of justice vs. physician responsibility toward the patient.  The physician told the patient they were going to consult with the transplant center about their managment.  After being contacted, the transplant  center asked for the patient to be transferred to them so they could manage and advise the patient.

 

Jamieson      5 Slide Follow Up

 60yo female with altered mental status.  Patient found wandering thru neighborhood by a neighbor.   Patient had anion gap acidosis.  INR was 3.2.  ABG showed respiratory alkalosis and metabolic acidosis.   ASA level was 106.2.

ASA toxicity clinical picture: tinnitus, vertigo, altered mental status,  nausea/vomiting, hyperpyrexia, pulmonary edema, coma.   Mixed acid-base disorder with respiratory alkalosis and anion-gap metabolic acidosis. 

 Treat with charcoal if it can be administered safely  and bicarb drip to alkalinize urine.   Indications for dialysis: coma, altered mental status, cerebral edema, pulmonary edema, renal insufficiency, clinical deterioration, or  serum level >100.   Avoid intubation if possible.  Mechanical ventilation frequently can’t match human minute ventilation. 

 

Meyers    5 Slide Follow Up

42 yo male with altered mental status and icteric sclera/jaundice.  Labs showed markedly abnormal LFT’s.   Lactate was 5.9.  Prolonged INR.  Utox was positive for opiates.  Vitamin K was given.

Patient admitted to ICU.  Labs showed both Hep B and C. 

 

*Hepatitis Serology (google image)

 

70% of patients with Heptitis B have a benign anicteric course.  0.5% of patients will have fulminant liver failure.   This patient fell into this latter group.

 

Entecavir is a nucleoside analog that treats acute hepatitis B with severe features (INR>1.5 or bili>10, immunocompromised, elderly, pre-liver transplant)

 

 

 

Conference Notes 5-7-2014

Bolton     GI Emergencies Study Guide

 

Erythema nodosum is associated with inflammatory bowel disease. 

 

Most common cause of bacterial diarrhea presenting to the ED is campylobacter.  Viruses are still the most common cause of diarrhea overall.  You can use anti-motility agents to improve patients’ symptoms of diarrhea. 

 

Pepto bismal taken QID and prophylactic antibiotics can reduce the incidence of traveler’s diarrhea but it is not recommended in a normal healthy traveller.   Resistance to Cipro is increasing when used to treat traveller’s diarrhea.  Elise comment: Is cipro still the drug to take with you on a trip?  Molly’s response: yes it is still first line for traveller’s diarrhea .   Elise checked the CDC website and indeed, cipro is still recommended for first line management of traveller’s diarrhea.

 

Emphysematous Cholecystitis is a rare but dangerous disease process.  It requires aggressive resuscitation and surgical management.  ED Antibiotics need to cover anaerobes and gram negatives.

 

 

*Emphysemaous Cholecystitis  (google image)

 

Toxic Megacolon needs to be treated with IV fluids, broad spectrum antibiotics, and IV steroids.   Probably prudent to discuss steroid administration and dosing with GI consultant.

 

 

*Toxic Megacolon  (google image)  Note large width of tranverse colon and thumb-printing

Epigastric pain that improves with food and worsens in the middle of the night suggests PUD.   Epigastric pain that worsens with food suggests gastritis

 

Crohn’s disease carries risk of retroperitoneal abscess and fistula.  Faculty comment: Great teaching point, this could be missed easily in clinical practice. Molly discussed a case in a patient with history of crohn’s and vague back pain and limp who had a psoas abscess.

 

*Chron’s disease with psoas abscess  (google image)

 

Porcelain gallbladder increases patient’s risk of gallbladder cancer.

 

Sonographic criteria for cholecystitis: wall thickness >3mm, pericholecystic fluid, CBD >7mm, positive sonographic murphy’s sign

 

*U/S of Cholecysitis  (google image)

 

 

Flagyl is first line therapy for C-Diff colitis.   Oral therapy is preferred.   If C-diff recurs you should initially attempt a second course of Flagyl and if that fails oral vancomycin is the next choice   C-Diff is a toxin mediated diarrhea.   C-Diff can develop even after a short course of antibiotics.

 

H.pylori irradication decreases the incidence of recurrent PUD down to the 15% range.

 

Crohn’s disease lesions tend to extend through all layers of the bowel.  UC lesions tend to be limited to the mucosa.

 

15% of cholecystitis is acalculous.  More common in elderly patients and ICU patients.

 

Coins in the esophagus will lie with the coin “en face” on the PA view.   In kids, the coin will get stuck in the proximal esophagus in the area of the cricopharyngeous (usually around the calvicles).   Button batteries in the esophagus have even more urgency to have them removed.  Resident comment: I have seen two button batteries removed from the esophagus and both children had burns in their esophagus within 1.5 hours.

 

 

*Coin in the esophagus (google image)

 

Infectious complications of pancreatitis usually develop 5-15 days after onset of pain.

Epiploic appendigitis is caused by torsion of an epiploic appendage (basically an epiploic appendage is a stalk of blood vessel/tissue/adipose/serosal surface hanging from the colon and projecting into the peritoneal cavity).  Treatment is supportive and it usually resolves in about 2 weeks.   Patients do not need surgery for this disease process.  Clinically it can look very similar to appendicitis.

 

*epiploic appendage  (google image)

 

*Ranson Criteria for Pancreatitis (google image)

 

Boerhaave’s syndrome: full thickness perforation of esophagus, usually due to vomiting, requires broad spectrum antibiotics and surgery, it carries a high mortality.

Ogilvie syndrome is a colonic pseudo-obstruction.  Treat with neostigmine and or erythromycin ( both stimulate colonic activity) and/or decompression with colonoscopy.

 

*Ogilvie syndrome (google image)

Most common cause of esophageal perforation is iatrogenic.

Most common cause of small bowel obstruction is adhesions.  #2 is hernia.  Girzadas comment: If a patient has a small bowel obstruction and they don’t have surgical scar on their abdomen, look for a hernia.  I have made the mistake of admitting a patient with an undiscovered incarcerated inguinal hernia.

Mesenteric Ischemia: CT abdomen/pelvis with IV contrast has 64% sensitivity for mesenteric ischema..  CT angio is diagnostic modality of choice with higher sensitivity than usual CT .   Mortality of mesenteric ischemia is 50% if diagnosed in first 24 hours

 

Febbo/Gupta          Oral Boards

 

Case 1.  84yo male presents with syncope and hypotension.   Patient is uncomfortable and is incontinent of stool.   EKG shows paced rhythm with sgarbossa criteria for STEMI.  CXR shows very large heart.   Diagnosis:  Cardiogenic shock due to Inferior wall MI           Critical actions:  Identify shock,  fluid resuscitation,  pressor support, cath lab activation.    Gupta comment: use ultrasound at the bedside to rule out other causes of shock such as pericardial fluid,  AAA,  tension pneumothorax, valve rupture,  PE, wall rupture.

 

Case 2.  24yo male presents requesting drug detox.  Pt has low grade temp.  He also feels sob and fatigued and has a cough.   On physical exam patient has oral thrush. Diagnosis:    PCP pneumonia         Critical actions:  Recognize clinical picture of HIV with cough/dyspnea, treat with Bactrim and steroids.   Gupta comment: ABG with Po2 <70 indicates need for steroids.

 

Case 3.    25yo female presents with dyspnea.  Patient has history of pyoderma gangrenosum      Diagnosis:   Methemoglobinemia due to dapsone treatment pyoderma gangrenosum         Critical actions: methylene blue administration.    Andrea comment:  If a patient taking dapsone has a G6PD deficiency, methylene blue can worsen the hemoglobinemia.   If a patient with methemoglobinemia from dapsone has anemia consider that they could have a G6PD deficiency and consider giving them a transfusion prior to giving them methylene blue.

 

Carlson    Anticholinergics and TCA’s

Anticholinergics predominantly affect the muscarinic receptors rather than the nicotinic receptors.  

Anticholinergic toxidrome: Blind as a bat, red as a beet, hot as Hades, dry as a bone, mad as a hatter., blurred vision, vasodilation, elevated temperature, dry skin/mouth/mucous membranes/arm pits, urinary retention, CNS depression/excitation/hallucinations.

Less common features include seizures, arrhythmias, coma, hypotension, rhabdomyolysis.

In patients with anticholinergic symptoms from antihistamine overdose, check also for acetaminophen overdose.  Many medications contain combinations of Benadryl and Tylenol.   Also check for ASA.

Anticholinergic Overdose Management: Manage agitation with benzodiazepines.   Avoid beta-blockers and calcium channel blockers.  Treat QRS prolongation/WCT with bicarb or lidocaine.  Physostigmine is a pure antidote to anticholinergics.  Physostigmine is a carbamate which is a reversible acetylcholinesterace inhibitor.  Average duration of physostigmine is 60 minutes but most anticholinergic drugs have an effect for several hours.  So ,  if patients improve with physostigmine they need to be observed for 4-6 hours to see if the anticholinergic symptoms recur. Indications for physostigmine are pure anticholinergic overdose and: coma, intractable seizures, severe agitation, or symptomatic narrow complex tachydysrhythmia.  There is still a lot of concern about giving physostigmine.  There are 1980’s case reports about fatal outcomes when given in setting of TCA overdose.   There has been some revisionist thinking about physotigmine lately and now it is considered more user/patient friendly.  Still, avoid it in the setting of TCA overdose, poly-drug overdose and with wide complex tachycardia.

Recent case report discussed the fact that topical atropine eye drops have 150 mg of atropine in a full bottle.  The bioavailability of topical atropine drops is 60%.  So this has potential for serious toxicity and possibly as a Mcgyver type antidote in the hospital if we ran out of regular atropine.  Resident comment: If there was a nerve gas exposure to a large group of people and the hospital ran out of atropine, you could have patients drink atropine eye drops.  Andrea agreed this was a reasonable idea.

Night shade is an anticholinergic plant.  It has a bunny ears leaf.   Mandrake is another anticholinergic plant.

 

*night shade leaf  (google image)

 

TCA’s are “dirty drugs”, they have multiple effects on the body.  Anticholinergic, alpha blockade, gaba blockade, Na and K channel blockade, H1/H2 receptor blockade effects.

Clinical picture: Tachycardia, QRS prolongation, Terminal R wave widening in AVR, brugada patern, QT prolongation, bradycardia/asystole,  hypotension, seizures, coma,

 

*TCA EKG  (google image) note wide terminal R wave in AVR, tachycardia and widened QRS 

Treatment for TCA overdose:  Sodium bicarb is indicated for QRS >110, marked acidosis, refractory hypotension, cardiac arrest.   Push 1 amp at a time.   You can also hang a bicarb drip.  Be sure to monitor the patient’s potassium. Second line antiarrythmic is lidocaine (1b agent). You want to avoid 1a and 1c agents.  

 

*antiarrythmic classification (google image)

 

Seizures are frequently self limited  in TCA overdose but first line treatment is benzodiazepines.  Second line treatmet for seizures is propofol. There are some case reports suggesting intralipid may be helpful.   Avoid flumazenil, physostigmine, 1a/1c antiarrythmics and beta/calcium channel blockers in TCA overdose.

 

TCA overdose disposition: If asymptomatic after 6 hours, they are medically clear for admit for psychiatric evauation.  If patient has persistent tachycardia, admit for OBS.   Any other signs admit the patient to the ICU.

 

Beckemeyer     Seizure Management

 

First line: Lorazepam.  Second line: phenytoin.  Third line: Phenobarb or Keppra or Valproic acid.

Intubation may be indicated when giving multiple anti-seizure meds.  Especially when using benzo’s and phenobarb.   If you have to use a neuromuscular blocker for a prolonged time you may need to have the patient on continuous EEG monitoring.

Consider pyridoxine for refractory seizures (INH overdose antidote)

Purple glove syndrome can occur from multiple doses of phenytoin in elderly patients.  This is not from a IV malfunction or extravasation into the soft tissues.   Harwood and Elise comment: This is likely due to propylene glycol.

 

*purple glove syndrome  (google image)

 

Febrile seizures: Occurs between 3months and 6 years of age, generalized seizure, last less than 15 minutes. Child is back to normal shortly after seizure.  Faculty discussion about work up indicated for febrile seizures: Most agreed that if child can be observed to be normal per parents within an hour after seizure they don’t need an LP.  They just need an appropriate work up for source of fever (exam, +/- UA and CXR)  In younger kids (<6 months) there may be some lower threshold to LP because it is more difficult to assess mental status/return to baseline.

 

Harwood comment: I don’t get any labs other than anti-seizure drug level in patients with known history of seizure disorder who present to the ED with an acute seizure.

Elise comment: Get a CT for trauma, fever, immunocompromise, change in seizure type, history of cancer,  warfarin use, or focal neuro deficit.

 

Paik    Follow Up  Lecture

 

49yo male with 2 months of jaundice and weight loss.  Pt has abdominal distension and hepatomegaly.  CT shows ascites and cirrhotic liver.  Patient was admitted from the ED.   20 days later he presents to the ED again.  This time he has hematemesis and hypotension.  Patient received protonix and octretide in the ED.  He was admitted to ICU. EGD showed portal hypertension with no active bleeding.

 

Management for Active hematemesis: Protonix, Octreotide, IV antibiotics, infusion of blood products,  vitamin K,  consider sangstoken-blakemore tube, consult GI consultant to scope patient emergently and or consult IR to perform TIPs procedure.

Dr. Paik discussed the technique of placing the Blakemore tube.   A key point is that hanging a bag of saline from the outer portion of the tube will apply adequate pressure on the stomach balloon.  Bonder comment: IV antibiotics have a low NNT for improving patients with upper GI bleed.  IV ceftriaxone is a reasonable choice but probably any antibiotic coverage of gram negatives is effective

 

 

Parker   Follow Up Lecture

 

Pt presented with respiratory distress due to aspiration pneumonitis.  Pt was in hospice but was placed on bipap in the ED.   CXR showed cardiomegaly and vascular congestion.  Pt improved clinically and bipap was discontiued.   Pt worsened and bipap was re-started.  Pt stabilized and was admitted.  Pt deteriorated on the floor and family made decision to withdraw bipap treatment.   Pt died on the floor.  

Aspiration is primarily a chemical pneumonitis. 26% have superimposed bacterial infection. Do not treat with empiric antibiotics.  Provide suctioning and positive pressure ventilation as needed.  No steroids. 

Elise and Christine comment: Don’t give empiric antibiotics to relatively healthy patients with simple aspiration.  Treat only if patient develops CXR infiltrate with signs of pneumonia.   Add clindamycin to standard CAP regimen and Zosyn to standard HAP regimen.

 

 

Nevin      High Reliability Organizations

 

HRO’s operate under very trying conditions yet manage to have fewer than their share of accidents.  HRO’s attempt to make systems ultra safe

 

Facts about errors: everyone makes them, most serious events are due to systems or process problems.

 

Diagnostic Errors: Framing, obedience to authority, premature closure, and anchoring.   Healthcare workers need to work with a questioning attitude to avoid these errors.

 

Most health care workers make 6.8 errors per hour.   Most serious errors that reach the patient got through 9 potential stop points.  

Variation is the enemy of quality.

200% accountability= 2 team members continually cross checking each other.

ARCC=Ask a question, Request a change, voice a Concern, invoke Chain of Command.   This is a protocolized way of raising a concern.

SBAR is a structured way to hand off care: Situation, Background, Assessment, Recommendation.

Errors should not be a cause for blame but starting point for discussions/investigations for discovering what went wrong.

 

‘Why hospitals should fly” is a good book to read regarding patient safety/HRO

Dr. Nevin then discussed the new Rural EM elective rotation being offered at Bromenn Medical Center.

 

 

Conference Notes 5-1-2014

 Conference this week was the ICEP  Spring Symposium.   The symposium included a Research Forum followed by presentations on  the impact of the Affordable Care Act (ACA/Obamacare).   I would suggest you read the ACA section of these notes with an eye for the following themes:

1.     The nation’s current healthcare costs are generally agreed to be not sustainable

2.     Emergency physicians have a key role in managing the healthcare costs of both the their individual institutions and the nation overall.

3.     Emergency Medicine as a whole provides great value/low cost for the nation’s healthcare system.

4.     With the ACA, ED visits are expected go up

5.     Patients covered under the ACA may not fully understand the co-pays and coverage limitations that they have.

6.     Healthcare systems like Advocate are tasked with moving from a model of generating revenue based on the volume of patients seen and procedures performed to a model of effectively managing the health of their patient population.  

7.     Illinois and many states have increased coverage under the ACA by increasing enrollment in Medicaid.  By Jan 2015 at least 50% of the patients covered under the ACA Medicaid in Illinois will be in a managed care format.

8.     Illinois is among the worst states in the country for EM in general and medical malpractice in particular.

 

2014 Research Showcase

Christ Presentation by Natalie Htet: FEIBA for ICH and life-threatening bleeding due to warfarin coagulopathy.    FEIBA was more reliable and worked faster to reverse INR than FFP.  FEIBA had more thrombotic events than FFP.  Mortality was higher in the FEIBA group likely to more patients having care withdrawn in the ED.  Also in the FEIBA group there were more ICH’s than in the FFP group.  The FFP group was predominantly GI bleeders who have lower mortality than patients with ICH.

 

Dr. Htet presenting at ICEP

Stroger Presentation:  U/ S evaluation of Dyspnea.  They looked for B lines in the lungs, cardiac function and IVC.  They looked at diastolic dysfunction as part of the cardiac evaluation.  This standardized evaluation was less sensitive but more specific for diagnosing CHF than clinician gestault informed by exam, CXR and labs.   Discussion following the presentation involved the validity of the gold standard for diagnosing CHF in this study.

Northwestern Presentation: Firearm injuries in Chicago in relation to day of the week and weather.    More shootings occur on Friday & Saturday and with warmer weather (risk is 30% higher when temp is between 80-90).  It was estimated that 26% of the risk of gun violence is due to ecological (weather) factors.

 Rush Presentation:  At presentation to the ED, demographic factors and clinical information of the patient can predict disposition.   Earlier disposition decisions have been shown to shorten ED throughput time.  Presenters developed a computerized decision algorithm that would provide an admit prediction to the triage nurse or ED physician and bed control.   Elise comment: Not sure this is any improvement over physician gestault.

There were also poster presentations at ICEP

Dr. Beckemeyer with her study on how a new Triage Protocol for Abdominal Pain Reduced the Rate of Patients Leaving the ED Prior to Being Seen by a Physician

9 of the 35 Best Residents on the Planet.  Thank you to Dr. Iannitelli for this photo!

ICEP Update

 Illinois gets a D- on the National EM Report Card for access to Emergency Care and a D for our Medical Liability environment.   Poison Control Center funding is at risk..   Illinois is one of the bottom 10 states overall for emergency medicine.   Elise comment: The only thing that makes me feel alittle better about being in Illinois as an emergency physician is that overall most states are at a C or D level.

There are many problems with FOID (Firearm Owners ID Card) notification requirements for emergency care providers.   Technically every patient with a psychiatric complaint in the ED needs to be reported to the FOID agency.

 Choosing Wisely Campaign for EM:  EM as a specialty is advocating for the following 5 potential cost saving conversations with patients:

1. Avoid CT head for minor head injury.  

2. Avoid urinary catheters.

3. Don’t delay the palliative care discussion with patients. 

4. Avoid antibiotics and wound cultures for uncomplicated abscesses.  

5. Avoid IV rehydration for kids with vomiting and diarrhea without attempting oral rehydration first.

 

Gerardi  (ACEP President)  Affordable Care Act: ACEP Perspective

Most of the money spent on healthcare in the US,  stays in the US and benefits the US economy.  Elise comment: True, but the high costs are a barrier to care for many people.  We still need to work to lower costs for patients.

 

Cost of health insurance is increasing much faster than income growth in the US.

A common comment made in the media is that emergency care is very expensive.  This was discussed many times throughout the day.  We as emergency physicians have an obligation to inform others that Emergency Medicine is not expensive for the nation, rather it is a great value.  Value=Quality/Cost. Emergency care accounts for only 2% of the US healthcare costs. Emergency physicians provide the bulk of acute care to the under and uninsured. Emergency physicians cover 67% of the nation’s unscheduled visits and 50% of hospital admissions.

 Average spending on healthcare per person is much higher in the 65 and older group compared to younger ages.   Also, in the US, expenditures in this 65 and older age group exceed other nations by far.   The majority of healthcare spending occurs in a person’s last year of life.   70% of the population uses 10% of the healthcare.   1% of patients use 30% of the healthcare in the nation.  These are called triple threats, they suffer from 3 or more comorbid illnesses.

 ED visits continue to increase.  The number of hospitals and ED’s are decreasing.  Consequently, across the country, average ED size and # of visits have increased.  ED’s generally are now large, complex systems.

 Triple aim of healthcare reform: Better patient experience, better outcomes, lower cost.  This will take a collaborative, team-based, patient-centered effort.  

 Affordable Care Act (ACA) provisions:  Expand Medicaid eligibility, create insurance exchanges, cover dependents up to age 26, and there is a mandate for people to enroll.   Patients cannot be denied coverage for pre-existing illnesses, essential health benefits will also be covered.  These changes will likely result in increased ED visits.

 Threats to emergency physicians: You will be less likely to be an independent physician and more likely to be a hospital employed physician.  There is a risk of cuts to emergency physician reimbursement.  Minute clinics such as those at Walmart and Walgreens will take away business from primary care docs.  Free standing ED’s and urgent care facilities are drawing emergency physicians away from hospital based ED’s that care for the underserved. Nurse practitioners can practice without physician supervision.

 ED ‘s can provide immunizations and wellness services to help the hospital meet it’s quality reporting goals.  Patient wait times are an important measure to a hospital’s   quality success.

 ACEP has concerns about the Choosing Wisely Campaign (Campaign described above).  They want to be sure that emergency docs won’t get sued if they do these action items. They also want to be sure insurance companies won’t refuse payment if  an emergency physician acts counter to one of these recommendations.

 ACEP expects that ED patient volumes will increase due to the changes brought on by the ACA (Obamacare)

 In Illinois overall, inpatient admit days have decreased.  

 Illinois has the second highest level of payout for lawsuits in the Country.

 Bronze plan of the ACA has a $5000 deductible for emergency care.  This will be a unpleasant realization for many people. 

 Panel Discussion on ACA in Illinois

 114,000 Illinois residents in addition to current Medicare enrollees signed up for the ACA.  

Sticker shock is expected for ACA covered patients.  They will learn as they access the healthcare system that there are significant co-pays and limitations of coverage.

 If a patient has not kept up on their monthly payments for ACA insurance,  providers may not get paid.

 There may be some limitations to preventative care coverage and coverage for certain non-generic medications.

The emergency physician will need to be sensitive to costs when managing the patient covered by the ACA.

 Attempts at developing a state-based insurance exchange in Illinois have been stymied by the insurance and small business lobby.

 Triple aim again:  Improve population health, improve the healthcare delivery system, and lower cost.

 The ACA has unleashed the potential for disruptive healthcare delivery initiatives such as pharmacies providing healthcare or cable companies providing telemedicine.

 Patients are going to be on the hook for more of their healthcare costs.   Emergency Docs have to be very sensitive to the costs that our patients face.

 Hospitals are all looking to cut costs based on the perception that re-imbursement for providing care will be decreasing over the next few years.

 Hospital systems are changing their focus from acute care to prevention and wellness and long-term management of illness to improve their patient populations’ health.

 There is a lot of anxiety in the hospital community and healthcare provider community over how all the change in healthcare is going to shake out.

 Jan 2015  50% of Medicaid patients will be in a managed care environment. 

 On the positive side, there will be millions more patients in Illinois with some type of healthcare coverage.

 There may be some pressure on emergency physicians to provide care in a way to keep patients out of the hospital.  Emergency physicians will be key players in controlling hospital costs. 

 llinois is in the bottom 2 states regarding the medical liability climate.  This affects how willing emergency physicians are to limit testing and discharge patients. 

 The ACA provides no increased funding for residency slots and training. 

 There is still strong job security for emergency physicians.  Income security may have pressure  in the next few years.

 Mila comment: Are there means in the ACA to affect patient behavior to stay within their managed care environment?   The ACA provides for community healthcare workers who can help patients make good decisions as to where they seek care.

 There seems little political will to make any significant change in the medical malpractice environment in Illinois.

 Audience comment: The ED provides rapid diagnosis and acute treatment.  It is a US center of excellence for diagnosis.  It provides patients the ability of getting a quick answer to their symptoms and get back to work more rapidly.  This is an under-appreciated way that emergency medicine provides true value to our country.

 

 

 

Conference Notes 4-23-2014

Knight/Williamson      Oral Boards

Case 1. 42 yo female  presents with cough, fever, and tachycardia.  Pt had pneumonia on CXR and CT imaging, no PE.  Patient also had afib, a low TSH and a goiter.   Diagnosis was thyroid storm and pneumonia.  Critical actions: Treat with  propranolol,  methimazole is preferred over PTU due to PTU’s risk of hepatic failure, SSKI given at least 1 hour after methimazole, and glucocorticoids to block peripheral conversion of t4 to t3.   Andrea comment: give dexamethasone or hydrocortisone as your choice of steroid. It has somewhat better activity than prednisone or solumderol.    Christine comment: Don’t give CT contrast to a person with hyperthyroidism who has not received PTU or methimazole because that will actually ramp up their hyperthyroidism and potentially move them into Thyroid Storm territory.   Also don’t give two AV nodal blockers to a patient.  Giving a CCB for Afib and following that with a Beta Blocker for thyroid storm could cause severe hypotension or bradycardia.  If you figure out the patient has thyroid storm after you initiated treatment with cardizem for Afib maybe give esmolol because you can turn it off if you get into trouble.  You can also just use cardizem for rate control as this is the recommended agent for treating storm in patients with bad asthma.

 

Up to Date Reference: Although there are no data showing that patients do better clinically with one thionamide over another, we suggest PTU for the acute treatment of life-threatening thyroid storm in an intensive care unit (ICU) setting, where it can be administered regularly every four hours. PTU, but not methimazole, blocks T4 to T3 conversion, and there is some evidence that over the first few hours after administration, PTU more rapidly reduces serum T3 concentrations than methimazole [12]. However, because methimazole has a longer duration of action, and after weeks of treatment results in more rapid normalization of serum T3 compared with PTU, and because methimazole is less hepatotoxic, methimazole may be preferred for severe but not life-threatening hyperthyroidism. Patients started on PTU in the ICU should be transitioned to methimazole before discharge from the hospital

 

Case 2.  56 yo female with acute headache.  Pt has difficulty with vision in right eye.   Right pupil is midrange and non-reactive.  Right eye pressure was 60 mm hg. Diagnosis is acute angle closure glaucoma.   Critical actions:  Alpha agonist (Brimonidine),  Beta blocker (Timolol), Pilocarpine, Acetazolamide,  Mannitol, topical steroid, emergent ophthalmology consult.  Girzadas comment: Good mnemonic is E=M2C or Eye=miotics (pilocarpine, beta-blocker, alpha agonist), mannitol, and carbonic anhydrase inhibitor.  It doesn’t cover the steroid but for test purposes if you know the miotics, mannitol, and acetazolamide you will pass.

 

 Google Image of Angle closure glaucoma right eye 

 

Case 3. 16 yo male with Right hand pain due to punching another person in the mouth. A day or two has passed since the trauma and patient has signs of wound infection.  Patient has a fight bite over the 5th mcp joint.  Xrays are negative.   Critical actions: Copious irrigation, IV unasyn, get xray to look for fx/fb, emergent hand consult.  Human saliva has multiple organisms including strep, staph , fusobacteriumand eikenella corrodens.   Harwood comment: Tell the patient and family that the patient has a really bad problem that can lead to severe disability.  I prefer to admit all these patients.  Don’t ever close a fight bite primarily.  Splint the hand.

 

Google Image of Fight bite/Clenched fist injury with resultant osteomyelitis.  Finger had to be amputated eventually.

Rosen Reference:

Clenched-fist injuries, also called “fight bites,” are notorious for being the worst human bites. Inadequate initial management leads to significant morbidity. Misleading history, innocuous wound appearance, intoxication and lack of cooperation of the patient leading to inadequate examination, patient reluctance to admit the nature of the injury, delayed presentation, and inadequate exploration all may lead to mismanagement. Clenched-fist injuries are associated with the highest incidence of complications of any closed-fist injury and of any type of bite wound.[85]

The classic injury is a bite wound over the third MCP joint, but injury can occur over any joint. Soft tissue injury is apparent with possible extensor tendon injury and violation of the joint capsule. When the fist is subsequently opened, the bacterial inoculum is dragged with the extensor tendon and soft tissue proximally into the dorsum of the hand.[85] Presentation may be acute or delayed. Swelling, limited range of motion, erythema, and pain out of proportion to the apparent severity of injury are typical findings. Pain is more severe with range of motion.

Aggressive management is indicated with these injuries. Immediate consultation with a hand surgeon is advised. Analgesics, irrigation, tetanus, cultures, intravenous antibiotics, appropriate wound care, elevation of the affected limb, and hospital admission should be considered for all patients. Foreign bodies are excluded with radiologic studies and possibly exploration. Tendon injuries are ruled out with careful exploration. The hand should be splinted in the position of function. Pathogens usually are polymicrobial, with Staphylococcus aureus, streptococci, and anaerobes the predominant species. Multiple-drug regimens are recommended; amoxicillin-clavulanic acid or penicillin with a first-generation cephalosporin is commonly used.

 

 Girzadas           Behavioral Based Expectations  

 Crew Resource Management (CRM): Method of communication and team work utilized by commercial and military aviators as well as our anesthesia colleagues to minimize errors and maximize patient safety.   Key aspects of CRM are the leader sharing his mental model of the problem at hand and his plan for addressing the problem with all team members so there is a shared vision.  Clear communication using repeat-backs and clarifying questions and phonetic and numeric clarifications are the other key aspects.

 STAR=Stop  Think  Act  Review.  This is a CRM tool to slow down when there is time pressure and be sure the action you are preparing to take is correct.  An example would be to stop and review the patient’s allergies, PMH, Chief Complaint, and current clinical condition prior to ordering a medication.

 CLER visit is happening April 29th and 30th.  CLER=Clinical Learning Environment Review.   This is basically the RRC coming to see how the hospital is training residents.   They focus on the areas of error reporting, patient safety, hand-offs, fatigue management, quality, and duty hours.

 

 Htet            FEIBA for Reversal of Warfarin Induced Life Threatening Hemorrhage

 Feiba is essentially a 4 factor prothrombin complex concentrate (factors 2,7(activated),9,10).     It has smaller volume and faster/more reliable correction of INR than FFP. On the downside, FEIBA has prothrombotic properties.

In our ED, patients on warfarin with life threatening bleeding:   if INR>5 they receive 1000u of FEIBA.  If INR <5 they receive 500u of FEIBA

Median time to INR reversal to less than 1.5 was 47 minutes.   93% of patients achieved INR goal of less than 1.5.

Mortality for FEIBA patients was 39.5%   14% of FEIBA patients had thrombotic adverse events.  1 patient accounted for ¾ of all thrombotic adverse events.

The mortality rate for FEIBA is higher than the mortality rate for patients receiving FFP in our Medical Center.  However, analysis of the data shows that patients receive FEIBA very early in the hospital course (in the ED). Patients who got FFP received it much later in their hospital course due to the time it took to prepare FFP.  Thus the severely ill patients who would have received FFP died before receiving it. Severely ill patients receiving FEIBA were still alive in the ED to get the drug but still die in the ED or early in their ICU stay.  Patients who lived long enough to receive FFP were less ill and more likely to survive.  This survivor benefit for FFP likely accounts for the difference in mortality between FEIBA and FFP.

 

Hemming                   Patient Safety   Medication Errors in the ED

Medication errors are the single most preventable cause of patient injury.  They accounts for 25% of litigation against physicians.

77% of medication errors in the ED occur between the order phase and administration phase.

Multiple factors contribute to med errors: unfamiliar patients, multiple patients, interruptions, verbal orders.    Verbal orders are more prone to error than written orders.

Closed loop communication:  Sender gives order,  receiver performs a read back or call back of the order and sender confirms order.  This three part communication is part of Crew Resource Management discussed above and has been shown to decrease the rate of miscommunication.

 If patients are in the ED for a prolonged time, they  frequently don’t get appropriate repeated dosing of medications.

 Most common type of medication error is physician ordering error.

Avoid trailing 0’s when writing dosing. Example to avoid: 1.0 mg of dilaudid could be misread to be 10 mg of dilaudid   Standardize concentrations and dosing as much as possible.  Double-check  dosing .  

 For home medications, it is considered best to have one person administer the medications for a patient to avoid errors.    It is also considered best to as much as possible have one physician be prescribing medications for a patient .

Girzadas comment: When you are asked to give a phone order for an ED patient who you haven’t seen for a few hours and you are caring for multiple patients,  STOP and regain situational awareness of that patient by reviewing their allergies, medical history an chief complaint.   Regaining that situational awareness of the patient will help prevent medication errors.  This is the STAR technique noted above.

 

Transcutaneous and Transvenous Pacing Lab

 

 

 

 

 

 

Conference Notes 4-16-2014

Lovell                          Pulmonary Study Guide

 Placing a mini chest tube is the best answer for treating a large primary spontaneous pneumothorax.   Small spontaneous pneumo’s (measured as 3cm or less from apex of the thorax to edge of lung) can be treated with observation and oxygen  and re-xray at 6 hours.  Oxygen helps a pneumothorax resolve more quickly.  If pneumo is not increasing at 6 hours, pt can go home.  Pneumo’s may take 2-3 weeks to resolve on their own.

Pneumothorax Calculator on chestx-ray.com

 Pneumo’s in COPDr’s are different than primary spontaneous pneumo’s and probably should get a chest tube in the low 20’s French size.  Trauma patients with pneumothorax should get a standard larger chest tube.

 Gotta differentiate between blebs and pneumo's in COPDr's.  Below are Blebs/bullae in a COPD patient

 

Blebs in COPDr’s can be easily confused for a pneumothorax.  If in doubt, get a CT to clarify the diagnosis.

 

The definition of massive hemoptysis is >600ml over 24 hours.  For quick estimation of hemoptysis volume in the ED: mild hemoptysis is blood streaked sputum,  moderate hemoptysis is clots/fress blood, massive hemoptysis interferes with oxygenation.   Remember that TB can be a cause of massive hemoptysis and wear personal protective  gear when at the bedside of a patient with massive hemoptysis.

Tracheo-Innominte Artery Fistula:  Happens within several days to 4 weeks of tracheostomy placement.  Patient can have a relatively minor appearing sentinel bleed.  But the next bleed is going to be catastrophic.   Get a CT chest/neck and a bronchoscopy in the ED for sentinel bleed.  If the patient is having massive bleeding,  over-inflate the trach balloon.   If that fails to stop the bleeding, intubate patient from above, remove the tracheostomy tube simultaneously and  stick your finger into the tracheostomy site anterior to the trachea and use your finger to compress  against the sternum.     Teaching point: Recent tracheostomy and bleeding from tracheostomy  site consider Tracheo-Innominate Artery Fistula. 

Harwood comment: in NH patients with bleeding from trach site, if you see an obvious source from the skin then probably no danger.  If you don’t see an obvious soft tissue source of blood, then you gotta CT and get a Bronchoscopy .

 

 

Tracheo-Innominate Artery Fistula Rescue Maneuver

 

Risk factors for lung cancer in a patient with hemoptysis: male, age >40,  smoker, recurrent hemoptysis, and no bronchitis/infectious symptoms.

 

Most common bacterial cause of pneumonia in HIV patients: Strep Pneumo.

Also think about TB in HIV patients even if their CD4 counts are OK.  Think about pneumocystis pneumonia in HIV patients with pneumothorax.

 

 

Pneumocystis pneumonia and Pneumothorax

 

Pneumonia symptom complexes: Sudden onset, rust color sputum, rigors think strep pneumo. Post influenza think staph.    Alcoholic think Klebsiella.  Joint pain, bullae on TM, rash, sore throat think mycoplasma.

 

Aspiration pneumonia:   Don’t use steroids.  You need anaerobic coverage in addition to usual CAP coverage.   Add clindamycin to usual CAP coverage.  Moxifloxin is a second line option that can be used as single agen.   For HAP with suspected aspiration, the Zosyn in our standard HAP regimen will adequately cover anaerobes.

 

IGRA blood test can be used in place of PPD to identify TB exposure.  The result comes back within 24 hours.  It is thought to have similar sensitivity and specificity to a PPD test.  Consider for use in admitted patients to help identify more rapidly who needs to stay in isolation.  Could also this test use in outpatients to avoid need to re-evaluate pt’s PPD site.

 

Primary TB infection is usually asymptomatic.  Latent stage has positive PPD.   5% risk of progressing from latent stage to active TB in 2 years in immunocompetent patients.   5% chance of converting from latent to active TB during patient’s lifetime in immunocompetent patients.    A patient is contagious only during active stage of TB.

 

 

Scrophula=painless lymphadenitis due to TB or other mycobacterium.  In adults it is most commonly due to TB.  In kids it is most commonly due to other non-TB mycobacterium

 

Scrofula

 

Negro/Doherty            Trauma Lecture

 

60 yo male was pinned between a car and a brick wall.  Pt arrived to ED with tachycardia and cool extremities (clinically in shock despite BP being still over 100systolic).  Pt had open pelvic wounds including rectal wound that were bleeding. It turns out that patient was on Plavix to make matters worse.

Pelvic Xrays showed very wide diastasis of pubic symphisis

Only check the pelvis for instability once.  Don’t repeat  pelvic instability exam if you think the pelvis is unstable.  Moving the boney fragments can increase bleeding.  

 Pelvic binders are still recommended by ATLS but there is no animal study data to support pelvic binding. Consensus by Trauma and EM at this conference was that pelvic binding was still helpful.  Pelvic binding should be wrapped around the greater trochanters not the iliac crests.   Binding will not have an effect on arterial bleeding.  It is intended to tamponade venous bleeding.  External pelvic fixation has no benefit over pelvic binding.   Patients with hemodynamic instability from a pelvic fracture despite pelvic binding should go to IR for embolization.

 

Patient continue to receive IV fluids and O negative blood.   Patient had an open rectal wound from sharp boney edge which was packed.

 Patient became agitated and was intubated.    Have to be careful about hemodynamic collapse in this type of patient when giving induction drugs.  Give conservative doses of induction agents.  Choose the most hemodynamically neutral induction agents and be prepared to give push dose of pressors for hemodynamic collapse.

 Massive transfusion protocol was initiated.

 Tranexamic Acid inhibits fibrinolysis by blocking lysine binding sites on plasminogen.   CRASH-2 trial showed a decrease in mortality  from all causes (14.5% vs. 16%).   TXA was given.

 

TXA competitively inhibits fibrinolysis

 

Next decision is whether to go to OR or IR.

 Doherty comment:  This decision is more complicated than the algorithms make it seem.  The right answer for boards is go to IR.  However, if there is a delay to get IR mobilized,  there are arguments to take a very unstable patient to the OR.  The goal of management for both IR and OR is to occlude the internal iliac artery on the side of fracture.

 FAST scan can help with this decision.  If you can get patient to IR rapidly and there is no intra-peritoneal blood on FAST then patient should go to IR.  If FAST shows intra-peritoneal blood the patient should go to OR.   Doherty comment: This situation is one of the most critical indications for FAST scanning.   It’s a binary test: If blood on FAST go to OR.  If no blood on FAST, go to IR.  Prior to FAST, old school approach was an open supra-umbilical DPL to identify intra-peritoneal blood.

 

Doherty comment: If patient’s BP bumps up during IR procedure it is a sign of successful embolization of  the internal iliac or branch vessel.

 

Beckemeyer/Katiyar            Oral Boards

 

Case 1.  Young boy presents with vomiting and passed out.   Family members from Viet Nam were visiting.  One family member was taking medication for “lung problem”.   Patient began seizing in ED.   Diagnosis was INH poisoning.  Critical actions were to manage seizures initially with lorazepam and to give pyridoxine as an antidote to INH induced seizures. 

 

Case 2.  14 yo female passed out twice at school.  Pt had palpitations at gym class and passed out .  EKG shows long QT interval.  Patient arrests in ED.   Monitor shows torsades.    Critical actions: Defibrillate and give IV magnesium.  Could also overdrive pace to shorten QT interval.

 

Case 3. 42 y/o female with worsening headaches, 4 weeks post-partum. Has seizure in ED, treated with magnesium. Diagnosed with eclampsia. Critical actions- CT/MRI, control BP, magnesium.

 

Knight  Hemoptysis

 

Unfortunately, I missed the majority of this excellent lecture.

 

Brisk hemoptysis is more deadly than exsanguination.

 Herrmann comment: When attempting to intubate the hemoptysis patient, suction the airway and place the yankaur suction catheter  through the cords and then pass a bougie right next to the yankaur through the cords.  You can then pass an ET tube over the bougie

 

Hemming                CHF and Pulmonary Edema

 

2 most Common causes of readmission for CHF: diet, medication non compliance.

Best history components for diagnosing CHF: CHF, MI, CAD

Best symptom components for diagnosing CHF: PND, Orthopnea, Edema

Best PE components for diagnosing CHF: S3, HJR, JVD

 

High level discussion between Elise, Erik, Christine, and Harwood about the utility of LR’s on the above items.   There was no clear consensus to report.

 

Sonographic B lines correlate well with Curley B lines and Lung water scores.  It can reliably diagnose CHF.  3 or more comet tails is suggestive of wet lungs.   Girzadas comment:  U/S is very reliable for diagnosis CHF.

 

BNP can diagnose and prognosticate CHF.   Other causes of elevated bnp: age, renal failure, ACS, chronic lung disease, large PE, high output cardiac state. 

 

Treatment of CHF:  NTG is first line.  IV furosemide is second line but efficacy is unclear .   Ace inhibitor is third line; some caution because it can cause hypotension.  Morphine is ineffective and may result in respiratory depression and worsened outcome.  Morphine may be indicated in diastolic dysfunction with elevated BP.   O2 and Bipap/Intubation should be used as indicated.  Balloon pump should be used for mechanical cardiac complications (wall rupture,  valve dysfunction,  papillary muscle rupture) causing CHF .  Inotropes are not usually indicated for CHF management due to increased mortality.

 

Discussion of Furosemide in CHF:  Erik’s comment was to take the middle road, give IV NTG and give small dose of IV Furosemide.   Harwood comment:  Give high dose IV NTG initially, titrate back to lower dose after first hour or two and also give conservative dose of IV  Furosemide.   Febbo comment: Research showed no difference between high dose IV Furosemide or an IV dose of Furosemide consistent with patient’s usual oral dose.   

 

Harwood comment: If you have to intubate a CHF patient you are a failure as an ER doc.  A good ER doc should be able to pull the vast majority of CHF patients back from the edge using Bipap and NTG and Furosemide.   Andrea comment: There may be the rare patient you have to intubate and you shouldn’t feel bad about that.   The faculty agreed that in the hypotensive CHF patient they have cardiogenic shock and these patients should be intubated and given an inotrope and should have emergent cardiology consultation .

 

Burt                  Patient Follow-Up

 

55yo female with abdominal pain.  Hx of cholithiasis and HTN.  No previous surgery.  Exam showed RUQ tenderness.  Initial work up focused on possible cholecystitis.  Pt later told examiner that she had been “walking funny”.   Repeat exam showed abnormal gait.   Labs all normal.  U/S of gb shows gall stones but no wall thickening.   MRI of spine showed epidural  spinal cord compression from tumor.  Breast, prostate, lung, multiple myeloma, lymphoma, sarcoma, renal cell, melanoma are potential primary neoplasms that can cause epidural cord compression. Epidural neoplasm or abscess are very difficult diagnoses and are frequently missed on the initial ED visit.

 

 Arrows point to lymphoma causing epidural compression

Htet               Patient Follow-Up

 

60 yo male with altered mental status.  Patient had unusual headache for week prior.  Hx of HTN, CKD, cardiac aneurysm s/p repair on Coumadin.    In ED atient was intubated and INR was 3.5.  CT showed SAH with trans-tentorial herniation. 

 

 

Colored arrows point to subarachnoid blood. Black and white arrows point to normal calcifications

 

Pt was given FEIBA in the ED.  Neurosurgery placed ventriculostomy in ED.  Dilantin  was given.  IV versed given for sedation.   Patient was having seizures every 3 minutes in ED.  IV Ativan was ineffective for stopping seizures.  Second dose of Dilantin given.  Keppra 500mg IV also not effective (could have given 1000mg).  IV phenobarbital 20mg/kg was given and worked.  Phenobarbital binds to GABA receptors. It reduces cerebral blood flow and reduces cerebral metabolic activity.  Down side is patients can have @100 hours of sedation and  also have hypotension.

 

Class 2B recommendation: Prophylactic anticonvulsants can be given in the immediate post-SAH period. 

 

Status epilepticus: Class 1 recommendation=lorazepam and midazolam.  Class 2 recommendations: Keppra,   phenobarbital, phenytoin, propofol, and valproate.  

 

Harwood comment: For a patient in status epilepticus start with aggressive benzo’s and then just start trying any other anti-epileptic.    When you combine benzo’s and phenobarb, you will have to be prepared to  intubate that patient if they become apneic.

Erik comment: In patients with refractory seizures who are getting hyperthermic, consider starting  therapeutic hypothermia.

Conference Notes 4-9-2014

Permar/Ryan    Oral Boards

Case 1.  50 yo male with etoh intoxication.   Labs show marked hyponatremia (107).   Diagnosis is beer potomania.  Critical actions: Avoid hypertonic saline because patient has no acute neuro findings. Put patient in ICU.  Slowly increase sodium with oral fluid restriction and IV normal saline cautiously.   The cause of hyponatremia in beer potomania is lack of dietary solute that causes the kidney to retain fluid.   Average sodium in these patients is 108.  Nick Kettaneh comment: EM Crit suggests fluid restriction.  Girzadas agrees, I think fluid restriction with very cautious/minimal saline administration is indicated

 

Case 2.  4yo female with leg pain.   U/S shows small effusion of the hip.  Labs all normal.  Pt improved with po Tylenol/Advil.  Diagnosis was toxic synovitis.    Critical actions were to evaluate for septic arthritis, fracture, other boney problem, and leukemia.   Ibuprofen is preferred for this malady over acetaminophen.

 

Case 3.   19 yo female with fever and altered mental status. Pt is also pregnant and has abdominal pain.  (cruel twist to the case).  Pt had gone to a clinic for an abortion 2-3 days prior to ED presentation.  Diagnosis was septic abortion.  Critical actions: Resuscitation with IV fluids , IV abx,  labs including quantitative HCG/ RH status, pelvic U/S, OB consult for surgery.   Girzadas comment:  ceftriaxone/gentamycin is not adequate coverage for septic abortion.  You need MRSA and anaerobe coverage so Vanco/Zosyn would be a reasonable choice.  Alternatively you could add clindamycin to the rocephin and gentamycin.  PharmD comment:  You don’t really need MRSA coverage in this clinical situation but you do need anaerobe coverage.   Sanford recommends ceftriaxone/flagyl

 

Levato    Antibiotic Update

 Uncomplicated UTI’s : treat with cephalexin for 7 days or nitrofurantoin for 5 days.  Creatinine clearance cutoff for using nitrofuantoin is above 60.

 Community Acquired Pneumonia that fails outpatient macrolide or beta-lactam therapy:  Switch to moxifloxacin.  Do not use Vanco/Zosyn unless patient meets HCAP criteria.   Harwood comment: I use CAP (IV ceftriaxone/azithro ) antibiotics for this situation.   For community acquired aspiration pneumonia, add clindamycin to ceftriaxone/azithromycin.    Alternatively, moxifloxin could be used as single agent coverage community acquired aspiration. Moxifloxin has anaerobic coverage.

 Cellulitis: Basic cellulitis without complicating issues like DM, peripheral vascular disease, or healthcare facility associated infections can be treated with IV cefazolin or IV clindamycin.    IV cefazolin has much higher tissue levels than po Keflex so a failure on po Keflex can still improve with IV cefazolin. You don’t necessarily have to go to IV vanco for Keflex failures.   Avoid Vanco/Zosyn for run of the mill community-acquired soft tissue infections.   MRSA doesn’t usually present as cellulitis so cellulitis does not need MRSA coverage.   For early cellulitis in a diabetic, IV cefazolin is also a reasonable choice.   For more severe infections (diabetic foot) cefazolin is not adequate coverage.   

 Bactrim and Doxy  both have 99% effectiveness against community acquired MRSA.

 

Chan         Quality Committee Update

 Clinical judgment is required to determine if post-arrest patient will go to cath lab.   In patients with multiple co-morbidities EM and Cardiology  physicians will have to make a decision whether or  not to go to cath lab post-arrest.

New LBBB without typical angina symptoms does not go to cath lab.  You have to have a new LBBB and  a good history consistent with cardiac pain.

Please give antibiotics for pneumonia prior to clicking “Dr. Done”    CMS criteria look at antibiotics given in ED for CAP.

To give HCAP antibiotics, document immunocompromise or that patient was hospitalized for 48 hours in the last 90 days.

 FirstNet now has an ED Stroke Alert powerplan.   That’s the powerplan to use in the ED when managing a stroke patient.   Goal to administer TPA for stroke is 60 minutes door to drug time at least 50% of the time.  Door to CT goal is 25 min.     Harwood comment: It is important to streamline registration process so that we can order tests/CT scans more rapidly.   It would be optimal to have TPA prepared prior to needing it and dispose of it if not needed.   Chan response: We had @250 Code Stroke activations and 41 of those received TPA.  That would be a lot of TPA wastage.   Kessen comment: At my new job, they use a digital clock in the patient’s room to count down the time left to give TPA in stroke patients or get STEMI’s to the cath lab.

 Please follow Restraint Order requirements very closely.  Please time when the patient is actually placed in restraints.   Make sure the restraints placed on patient match the order entered in FirstNet.

 CAUTI = Catheter associated UTI’s.   You must document an appropriate indication for foley placement.  This is an Advocate-wide priority to decrease foley catheter use/infections.  Straight cath’s do not fall into this monitored category.    You can straight cath multiple times instead of placing a foley.  Multiple straight caths are preferred over indwelling foley placement.

 Burn Trauma: Have a low threshold to consult Burn Center.

 Pediatric Psych Patients: SASS worker does not have final say on whether a patient is admitted to a state facility.  MD can override SASS recommendation and SASS worker has to find a facility for patient admission.

 

Walchuk             M & M

 79 yo male with hyperkalemia and renal failure.  Hx of AICD, HTN, AFIB, CAD, CHF.

Vitals showed hypotension.  Exam showed dehydration.  CXR was neg.  EKG was paced.  K=7.5  Cr=9.2  INR=5.9

Initial treatment: Hyperkalemia management, IV fluids, Bicarb drip, foley placement.

Pt went up to floor and it was learned that patient was digoxin toxic.   Patient was transferred to the ICU and given digibind.

Digoxin blocks the sodium-potassium ATPase transport mechanism leading to increased intracellular calcium and increased contractility.

Digoxin increases vagal tone and slows done AV nodal conduction and decreases AV nodal automaticity.   Digoxin increases ectopic beats.

 

Clinical findings of digoxin toxicity: nausea/vomiting, lethargy, coma, nearly any dysrhythmia except rapidly conducted supraventricular arrhythmia.  Ectopic beats like PVC’s are common.   A common finding in digoxin toxicity is SVT with AV block.   Bidirectional tachycardia is virtually pathognominic of digoxin toxicity.  EKG findings with therapeutic digoxin levels are  scooped ST segments in lateral leads.   Hyperkalemia is common with digoxin toxicity.  The degree of hyperkalemia is a better predictor of mortality than digoxin level or EKG findings.

 Digoxin Effect

BiDirectional Tachycardia

Treatment for digoxin toicity is digibind.  Avoid class 1a antiarrythmics.   Do not use procainamide in the setting of digoxin toxicity.  

Calcium administration in the setting of digoxin toxicity is controversial.  There are a few case reports of death but most recent reviews show that it is generally safe to give.

 Indications for Digibind: life threatening dysrhythmia,  K>5,  chronic digoxin poisoning, Digoxin level >15.

Carlson and Harwood comment: Probably digibind was not indicated in this case.  The cause of hyperkalemia was likely mostly dehydration and worsened renal function.   The fact that the patient had a pacemaker gave an extra margin of safety.  Andrea would have kept the digoxin at the bedside.  She would have arranged for dialysis.   Andrea comment: As the emergency physician you have a responsibility to identify what meds the patient is on especially if the med list is not easily found.    Harwood comment: Get a digoxin level in patients with hyperkalemia or if patients have slow afib.

E. Kulstad          Study Guide    Cardiovascular

 Asa is one of maybe two drugs that has been shown to unequivocally reduce mortality in AMI.  The absolute risk reduction of ASA is 4%.  The other drug that has shown mortality reduction is thrombolytics.

New Research finding: Don’t give ASA pre-operatively (GI surgery, ortho surgery, gu surgery, cancer surgery, etc) to prevent cardiovascular complications.  There is no benefit in MI prevention.   Clonidine also does not work to prevent MI’s post-operatively.   Asa has increased bleeding risk with no benefit in MI prevention post-op so don’t give pre-op.

 Criteria for benign early repol: widespread st elevation, j point elevation,  concave up, notching of j point, concordant/prominent  t waves.   Girzadas comment: also has to be no reciprocal changes.  Erik response: True but that is not included in the formal definition.

BERP

Electrical alternans is kinda pathognomonic for pericardial effusion.  This ekg finding has low sensitivity and high specificity.   There was a discussion about whether this finding was truly pathognomonic.

Electrical Alternans (Not sure the difference between electrical alternans and bi directional tachycardia except bidirectional tachycardia is probably going to be wide complex.)

Tangential Quest for Knowledge:

Pathognomonic (often misspelled as pathognomic and sometimes as pathomnemonic) is a term, often used in medicine, that means characteristic for a particular disease. A pathognomonic sign is a particular sign whose presence means that a particular disease is present beyond any doubt. Labelling a sign or symptom "pathognomonic" represents a marked intensification of a "diagnostic" sign or symptom.  (Wikipedia)

 In medicine, the term sine qua non is often used in regard to any signsymptom, or finding whose absence would very likely mean absence of the target disease or condition. The test for such a sign, symptom or finding would thereby have very high sensitivity, and rarely miss the condition, so a negative result should be reassuring (the disease tested for is absent). 

In contrast, a pathognomonic sign or symptom is one whose presence would very likely mean presence of the target disease or condition. The tests for such signs are highly specific and very unlikely to give a false positive result. (Wikipedia)

So my take is:  sine qua non is highly sensitive and pathognomonic is highly specific.  

 

Stress testing: sensitivity is @80% and specificity is also @80%.  The outcome stress testing is measured against is coronary stenosis on angiogram.   If you identify a stenosis on stress testing, there is no mortality benefit of stenting over maximal medical therapy (smoking cessation, lipid management, bp control, exercise, anti-platelet therapy, weight control).  

 

New sea-change in cardiovascular management this month with the JNC raising BP goals for older patients to <150/90 and AHA recommending decreasing frequency of lipid testing.

 The only clear indication for an angiography/stenting to decrease mortality is in the STEMI patient.

 The classic diagnosict triad of aortic stenosis is chest pain, dyspnea, and exertional syncope.  It is the most common valvular lesion in the US and about 3% of patients over age 75 have this problem.  In patients with syncope and AS the 3 year mortality is 50%.   In patients with dyspnea and AS the 2 year mortality is 50%.  Treatment is surgery or now there is a Transcatheter Aortic Valve Replacement Therapy available.

 Heparin has 0% risk reduction for death in the setting of ACS.

Purnell        Safety Lecture     EKG’s

 EKG’s frequently are ordered by persons other than the physician and they can be lost or not seen by the physician.   EKG’s are frequently not in a patient’s chart when we go in to see a patient.  We can forget to check the patient’s name on the EKG.

Carenet plus has EKG’s stored electronically and the ED EKG’s can be accessed while the patient is still in the ED.

  Harwood comment: A reasonable system fix would be to have an EKG image or interpretation linked to the tracking board similar to lab and imaging results.

 Energetic discussion of solving these EKG problems ensued.

 

 

 

 

 

 

 

 

 

4/2/14 Conference notes

Dr. Girzadas is at CORD, poor substitution by C Kulstad

8-9: CV Study guide by Dr. C Kulstad

9-930: Safety lecture by Dr. Balogun

Presentation of 4 cases with various chief complaints united by delayed urine testing. 89% of patients with GI/GU complaints have documented UCG at ACMC. Takes 2.5-4 hours after registration to get UCG performed here.  Reasons UCG is delayed- patient can’t urinate, sample there but not processed, not considered important, or ED busy (staff overwhelmed, sample lost).

Important to avoid delay in imaging, psychiatric placement, medication choices.

Ideas for improvement 1) expand and enforce triage UCG orders 2) order UCG for patient of childbearing years who might need medications or imaging before you see them 3) talk with nurse/tech 4) give pt urine cup and instructions and/or perform urine hcg 5) straight cath when UCG is critical to management

Pushback- physicians don’t need to take on additional roles unless emergent need. Compromise- give cup and instructions then notify tech/RN that pt is giving sample

930-12 U/S lecture and workshop- Dr. Lambert

Lambert finds 1st trimester pelvic ultrasound most meaningful ED ultrasound  for patient management.

Anatomy reminders- pelvis tilted 45 degrees anteriorly when upright. Just behind symphysis pubis will be the bladder. Uterus tends to be horizontal. There is peritoneal reflection between bladder and uterus- not normal to have fluid here. Another peritoneal reflection between uterus and rectum (aka pouch of Doulgas)- very common to have small amount of fluid here in normal patient.

 

Ovaries are next to external iliac vessels, just anterior to ureters.  Bowel gas can obscure visualization of uterus, helps if bladder is distended.

Yolk sac should be visible by 5 wks. Often see cardiac flicker just afterwards, at  5-6 wks.

Ways to image pelvis- Transabdominal- place probe just above symphysis pubis. Use bladder as acoustic window. Less invasive and good field of view. Lower frequency probe means lower image quality. Uncomfortable to press on distended bladder.

Obtain transverse and sagittal views. Sagittal- indicator points to umbilicus. You will see bladder (triangular at top of screen) then uterus (pear shaped). At inferior part of screen- see vaginal stripe towards left screen. Should see endometrial and vaginal stripe in one view, that shows you are in midline. Transverse- Indicator to right. Top of screen will show bladder (rectangular) with uterus (oval) posterior. Ovaries may be seen inferior to bladder at edges of uterus (often are not seen).  

 

Sagittal                                                                            Transverse

 Transvaginal- better because probe is closer to organ of interest, and the high frequency probe gives better images. Not than uncomfortable, especially compared to pressing on full bladder. Get wide field of view but not as much depth. Anatomic relationships can be confusing. Obtain sagittal and coronal views.

Sagittal- indicator up. Think of flipping the sagittal transabdominal view 90 degrees counterclockwise. See long axis of uterus with cervix towards right of screen (opposite of indicator side). May need to tilt probe by moving back of handle down (tip of probe to ceiling). Bladder may be visible at top, left of screen.

Coronal- see slices of uterus from cervix to fundus in short axis- back of handle down to see fundus. May get better images by backing probe out a little bit (make sure you don’t allow air gap which would degrade image quality a lot).

Ovary – 2 x2 x3 cm- should be oval and have peripheral follicles, may have to slide probe lateral to cervix and a bit deeper. They should be inferior/medial to iliac vessels (can use color flow to identify vessels).

Cystic structure on ovary in pregnant patient is corpus luteum- usually a couple of cm but can get up to 6 cm. Starts regressing at 6 wks

Endometrial stripe- hyperechoic inner part of the uterus, has 3 layers. Decidua is the same thing as endometrium, just in a pregnant patient. Double decidual sac is endometrium over embryo, seen at 4 wks.

At 5 wks, see yolk sac which looks triangular inside decidual sac.

60% of u/s done in ED for r/o ectopic in first trimester will show IUP clearly. Greatly decrease time to patient dispo. Improve patient satisfaction as you spend more time with patient. Plus, unstable ruptured ectopic patients can’t go to ultrasound.

Diagnostic criteria for 1st trimester ultrasound

Live IUP- gestational sac at least 5 mm internal diameter within the endometrial echo of uterus with 1) fetal pole and 2) heart beat

IUP- same criteria but without cardiac activity.

Abnormal IUP- same criteria but 1) gestation sac > 10 mm w/o yolk sac or 2) gestational sac > 16 mm and no fetal pole or 3) obvious fetal pole w/o cardiac activity

Extrauterine gestation- gestational sac at least 5 mm internal diameter- outside endometrial echo and one of the following 1) yolk sac or 2) fetal pole.  This is why landmarks are so important! Ectopic pregnancies often look like they’re in the uterus if you do a cursory ultrasound.

No definitive pregnancy- 1) normal uterus or 2) sac that isn’t big enough yet or 3) gestational sac with yolk sac or fetal pole.

15-20% of ultrasounds end up being no definitive IUP. About 30% of those will end of being ectopic- rate increases with free fluid or mass seen. Must have good follow-up arranged.

 

Hands-on practice

Conference Notes 3-26-2014

Hemming/Coghlan           Oral Boards

Case1.   65yo female with vague chest discomfort.   EKG shows posterior STEMI. However,  patient is severely anemic due to GI bleeding.  Rectal exam needed to be done to identify the GI bleed.  Instead of standard treatment and transfer to Cath Lab, correct management was to avoid heparin/Plavix , give O2 and transfuse.

 

Harwood comment: Posterior STEMI’s in general should go to the cath lab.  There is no debate about this as there is with LBBB.  This patient, however, was severely anemic so needed PRBC’s not anticoagulation and cardiac cath.

 

Case 2.  25 yo Female  who is 28 weeks pregnant presents with headache. PMH of DVT.  Pt is not hypertensive.  CT shows cerebral venous thrombosis.  Initial treatment is systemic anticoagulation.   DDX for pregnant patient with headache: pre-ecclampsia, ccerebral venous thrombosis,  idiopathic intracranial hypertension,  carotid or vertebral artery dissection, and SAH.   Dense triangle sign on a non-contrast CT is a marker of cerebral venous thrombosis.

Dense Triangle Sign

Empty Delta Sign is sign of Venous Thrombosis on contrast CT

Harwood comment: Consider CO toxicity in headache patients.  Ask if they have a working CO detector in their home.   Reglan (Class B) is preferred over prochlorperazine (Class C) in pregnancy.

Elise comment:  Re-frame this case in your mind as a pregnant woman with history of previous dvt and now has a headache.  This should point you toward a clotting related disease (cerebral venous thrombosis)

 

Case 3.   35yo male with eye pain.    Flourescein staining shows herpes keratitis.   Treatment is viroptic drops.

Girzadas comment:  Code word on tests for herpes keratitis is “dendritic pattern on cornea”.

Herpes Keratitis 

 

Williamson       AICD’s/Pacers

 Thanks to Kelly for sharing her images from her slides for the Conference Notes!

Pacemaker leads pass through the subclavian vein into the right atrium and into the right ventricle.

 Pacemakers have two primary functions: Sense cardiac activity and provide an electrical stimulus to the myocarium.

NBG Pacemaker Coding:  First letter  identifies chambers paced.   Second letter identifies chambers sensed.  Third letter identifies response to sensing. (Triggered, Inhibited, Dual)  Example: VVI= ventricular pacemaker,  ventricular sensing,  Inhibited by ventricular native impulse.     DDD=atria/ventricle paced,  atria/ventricle sensed,  both atria/ventricle can be triggered or inhibited.    DDD is the most common type of pacemaker placed currently. 

Erik comment: There are new pacemakers that are wireless.  Harwood and Erik discussed some of the facets of wireless pacemakers.  Basically there are no complications inherent with wires in the body. 

 Hiccups or muscle twitches in a patient with a pacemaker can be a sign of lead migration.

 Most paced ekg’s will have a LBBB appearance due to pacer wire being in the RV.  The RV is depolarized first hence the LBBB appearance.

Harwood comment:  You frequently have difficulty visualizing pacer spikes on EKG’s in the ED because our ED EKG machines are all set to reduce electrical/movement interference maximally and this will hinder visualizing pacer spikes.  The EKG machine’s internal computer will be able to pick up the pacer spikes and the EKG read will say paced EKG even if you can’t see the spikes.

 

There was a discussion among faculty about where to place a central line in a patient with a pacemaker.  The consensus was: don’t place a line in the IJ or SC sites on the ipsilateral side of the pacemaker.   Either the wire won’t pass or you risk dislodging a clot or potentially tangling the wires.

 

Placing a magnet on a pacemaker turns off the sensing function.  It puts the pacemaker into an asynchronous mode.   It does not turn off the pacemaker.  There are not a lot of uses in the ED for this. Probably it would be useful if the pacemaker malfunction is over sensing.  The magnet is in the inventory room in a locked cabinet.  There is a small risk to changing a pacemaker to asynchronous mode.  It could theoretically cause v-fib, but this would be rare.

 Asynchronous pacing causing V-fib

4 types of Failure:  Failure to Capture (spikes not followed by  P wave or QRS),  Failure to Pace  (no spikes) , Failure to Sense (inappropriately placed spikes that are falling within native rhythm),  or Pacemaker Caused Tachycardia (Runaway pacer which is pretty rare)

 Failure to Capture

Failure to pace

 

Failure to Sense 

Purnell and Herrmann comments: Hyperkalemia can adversely affect pacemaker’s ability to capture or pace.

Pacemakers can develop pocket or lead infection and this can be a very serious infection.  20-30% thrombosis rate associated with pacer but these are most commonly asymptomatic.

Sgarbossa Criteria are helpful in identifying  STEMI in a patient with a paced EKG.

 

AICD Issues:  You can get a small troponin leak from an AICD shock.   Patients are instructed normally to not go to ER if they are shocked and asymptomatic.  Asymptomatic patient with a single shock can go home after discussion with cardiologist.   Pt’s who receive multiple shocks or have chest pain need to be evaluated in the ED and most likely admitted.    Electrical storm is 3 or more shocks in 24 hours.  Electrical storm has a high mortality due to deteriorating malignant arrhythmias.

If you determine that the AICD is giving inappropriate shocks, place the magnet on the AICD  for 30 seconds to turn off the AICD.  If you do this, put the transcutaneous  pads on in case the patient has V-Fib.    Taking the magnet off the AICD does not turn the AICD back on.  You have to take the magnet off and then replace the magnet on the pacer to 30 seconds to turn the AICD back on.

 Asynchronous Pacing causing V-fib

Frazer      Trauma Lecture  Fire Related Injuries 

Case was presented of a patient rescued from a house fire with severe acidosis and markedly elevated CO level.

Smoke inhalation has a 40% mortality.  Smoke inhalation is a greater predictor of mortality that BSA burned.

Dry air does not conduct heat well below the vocal cords.  So airway burns from dry air are predominantly in the upper airway.  Steam inhalation conducts heat 400X more  than dry air and can severely burn lungs.  Most people with steam inhalation die before getting to the hospital.

If you see evidence of significant airway burn, intubate early to avoid airway obstruction due to worsening edema. Faculty discussion on the significance of singed facial hair:  Consensus was that if patient only has singed eyebrows/hair/beard and were not in an enclosed space fire they are not at risk for airway obstruction.  The example given was someone lighting the barbeque who has a flash burn is not at risk for severe airway edema.  Patients rescued from an enclosed space and/or if they have any other signs such as pharyngeal edema, stridor,  or carbonaceous sputum,  intubate them early. 

 

CO poisoning:  Displaces O2 from hemoglobin,  shifts the oxy-hemoglobin dissociation curve to the left , binds myoglobin,  and inhibits cytochromes.  Symptoms include headache, nausea, syncope, seizure, arrhythmia, death.  Identify CO toxicity in the ED with ABG with CO-OX or finger probe device.   High normal cut off for the finger probe is 6.6%..   Anything over that get an ABG or VBG.  Elise and Christine comment:  VBG is a reliable and less painful alternative to ABG for identifying CO poisoning.  Hyperbaric treatment is indicated for CO poisoning with a level over 25%, pregnancy, syncope, neuro symptoms, cardiac ischemia, acidosis. 

 CO shifts the Oxyhemoglobin Curve to the Left

 

Hydrogen Cyanide  (HCN) Poisoning:  byproduct of burned plastics, nylon, cotton, wool, polyurethane.  It is a cellular poison that blocks cytochrome function.   ABG shows anion gap acidosis and lactic acidosis   Treat empirically for patient with smoke inhalation and high lactate or anion gap acidosis.  Treatment is hydroxycobalamine  5g IV for adults and dose can be repeated once.  Pharmacist comment:  This is a drug that is stocked in minimal amounts.  You may have to call other hospitals or the poison center if you have multiple patients come in who are all poisoned by HCN from a fire.  Second choice is sodium thiosulfate.   Andrea comment:  HCN is pretty rare in the ED because the vast majority of patients die at the scene.

 

Elise comment:  If you chose to resuscitate a victim of a house/industrial fire and they have significant acidosis you pretty much are obligated to give them empiric hydroxycobalamine.

 

Carlson   

Antidote for acetaminophen toxicity is n-acetylcysteine  (NAC).  It regenerates glutathione, free radical scavenger, detoxifies NAPQI, and detoxifies parent compound.   150mg/kg IV,  then 50mg/kg at 4 hours, an 100mg/kg at 16 hours.

 

Anthi histamine toxidrome is tachycardia, fever, mental status change, dry mucous membranes, urinary retention.  Antidote is physostigmine.  Trade name for physostigmine is Antilerium.

 

Phenazopyridine (Pyridium or Azo) when taken in excess can cause methemoglobinemia.  Patients will have cyanosis and pulse ox is usually 85% whether they are on R/A or O2.   This is due to the way the pulse ox reads methhemoglobin via light wavelength.  Benzocaine (Ora Jel) can also cause methemoglobinemia.

        

Dextromethorphan is a semisynthetic derivative of morphine.  Acts at NMDA receptor and has dissociative effect.   It turns a drug screen positive for PCP.  Ketamine also turns a drug screen positive for PCP.  So if drug screen is positive for PCP, it could be due to  PCP, dextromethorphan, or ketamine. Treat the dextromethorphan overdose patient with benzos.

 

Caffeine binds to the adenosine receptor and can block adenosine’s effects.  In the setting of SVT with caffeine overdose, adenosine may fail.  For unclear reasons cardioversion  has also been reported to fail in the setting of caffeine overdose.  Calcium channel  blockers  or  benzos have been reported to be successful in treating  SVT in this setting.

 

Iron poisoining: treat if patient is symptomatic and level is over 350 ug/dl.  Treat all patients with levels over 500 ug/dl.

Percentage of iron:  Gluconate 12%   Sulfate 20%   Fumarate33%.  If patient ingests >60mg/kg  this dose will usually get you a level over 500ug/dl.  No need to get TIBC in the setting of Iron overdose because TIBC is unreliable.  Treatment is deferoxamine until acidosis clears. 

Elise comment: If patient has no gi symptoms in first 6 hours, they will not have a serious ingestion.  Example is a child with uncertain ingestion who mom brings in 4-6 hours later and is asymptomatic.  No blood testing for iron level needed.

Vitamin A toxicity can adversely affect the brain and the liver.

Goody’s Powder and BC powder for toothache contains ASA and can cause salicylate poisoning.  Bengay and Peptobismal and Alka-Seltzer all have ASA in it.  Pediatric beptobismal does not have ASA .   Blowfish tabs for hangovers, have a large amount of salicylate in them. Look for mixed acid base disorder to diagnose ASA toxicity.  Tinnitus is pretty reliably noted by patients with levels over 20mg/dl. Alkalinize urine to get ASA out of CNS.  

 

Burns       Case Follow Up

40yo male with acute abdomen and shoulder pain.  CT abdomen shows splenic rupture.  Pt had no hx of abdominal trauma.

 Atraumatic splenic rupture is rare and can be life threatening.  93% of atraumatic splenic rupture cases have a pathologic spleen.  Mortality is highest in patients age>40, hx of neoplasm, and splenomegaly.  

If patient with splenic rupture and intra-abdominal  bleeding is unstable, they go to OR.  If stable, they go to IR for splenic artery embolization.

Kehr sign is shoulder pain due to diaphragmatic irritation.

 

Bonder      Case Follow Up

55yo male with altered mental status.  Septic picture with fever/tachycardia/hypotension.   Patient was being treated for sepsis but no definite source was identified.   CPK was ordered and was 70,000.  Diagnosis was neuroleptic malignant syndrome due to new antipsychotic (Seroquel).  Patient responded well to bromocriptine.  Remember to consider non-infectious causes of fever in your differential. 

Conference Notes 3-19-2014

 

Chastain           Tales from First year out of Residency

 

Case 1:  Be cautious when disimpacting patients with severe constipation.   Prior to the procedure, take a thorough history looking for risk of bleeding.  Michelle discussed a case of severe bleeding following disimpaction because pt’s INR was very prolonged.  This fact was not known prior to disimpaction.    DenOuden  had also discussed a case several months ago in which a patient became very bradycardic due to disimpaction.

 

Case 2.  Elderly patient with shoulder dislocation.   Adequate sedation was obtained with propofol, but shoulder could not be reduced.   Second attempt with etomidate was successful.   No one present had a solid explanation for why etomidate would be more effective than propofol other than possibly pt was more relaxed.  Be prepared to alter your approach following failure of first attempt.

 

Case 3.  Trauma patient sedated with ketafol for hip dislocation.   Pt also received dilaudid prior to the procedure.   Pt had prolonged apnea during procedure.   It turns out pt had liver laceration and was not able to metabolize propofol due to liver dysfunction.    Attending discussion of why this pt had prolonged effects of ketofol:  Liver dysfunction due to trauma or genetic difference in metabolism of opioids and propofol.  Bottom line: get history looking for liver or renal dysfunction prior to administering sedation.  

 

Case 4.  Senior  with fever and altered mental status.  CXR shows right side infiltrate.  Treatment initiated in the ED for pneumonia/sepsis.   Attending who received patient in sign out  ordered a head CT (just to be thorough).  Subdural hematoma was identified.   You have to consider all possible causes of altered mental status.  In retrospect, pt had some bruising of shoulder indicating a fall and pt was on subQ heparin.  Faculty comments: No one routinely gets head CT’s in altered patients who most likely have sepsis.  However if there are signs of trauma and/or the patient is receiving anticoagulation, CT head should at least be considered.

 

Case 5.   46 yo male with chest pain and equivocal/concerning EKG (could be STEMI or Pericarditis).     Patient had risk factors for CAD.   EKG showed some evolution during patient’s ED stay. Bedside echo was unremarkable. Code STEMI was called and patient was cathed.   Pt had clean coronaries.   DX was pericarditis.  Clues favoring STEMI are ST depression, ST elevation lead 3>lead 2, Convex up ST elevation, new Q waves.  Clues favoring Pericarditis: PR depression,  TP depression (Spodick’s sign) Amal Mattou at AAEM when discussing this topic felt PR depression was unreliable.

Spodick's  Sign

  Kelly comment: You did the right thing.  Patients with evolving EKG’s  should always go to the Cath lab.  Elise comment: Bedside echo was a great idea.  If you see a pericardial effusion (pericarditis) or wall motion abnormality (STEMI) you have the diagnosis.

 

Case 6. 29yo female with sore throat and ear pain. Treated with Bicillin and discharged.  Next day came back with bilat peritonsillar abscesses.  Pt had bedside drainage by ENT and received IV antibiotics.  Make sure you inform patients to return if their throat is worsening .  Ear pain with sore throat may be a clue to early peritonsillar abscess.    Girzadas comment: There is no data showing that treating strep pharyngitis prevents peritonsillar abscess.  When you  care for a patient with straightforward pharyngitis, be sure to document in your chart that there is no peritonsillar abscess present.

 

Case 7.  Cardiac arrest.   Pt had enlarged RV on bedside echo.   Crash EPI drip:  1 mg of EPI in a liter of NS =1mcg/ml.   Put on a pump and start at 10ml/min and titrate up.   TPA was given for suspected massive PE.   Pt did not survive.  

 

Case 8.  65 yo  female with SOB.  Hypoxic & in extremis.  Exam showed Rales and lower extremity edema.   Bedside U/S shows B lines in lungs.  Diagnosis=CHF.   Pt was treated with BiPap but continued to worsen.   Michelle had to figure out that the default FIO2 on bipap at that hospital was 21% not 100%.  Respiratory tech had placed FIO2 at 21% the default level. Once the FIO2 was increased to 100% pt did well.   Michelle and Kelly comment: In community ED’s, nurses won’t necessarily notify you of the arrival of a sick patient.   You need to have global awareness of the room and ask the nurses if any new patient is sick.  Respiratory therapist may also not be attuned to the needs of severely ill patients.

 

Case 9.  70yo male with Hx of cancer and complaint of pleuritic chest pain and SOB.    Pt planned to have VQ scan to eval for PE and was signed out to Michelle.   CXR was reviewed and showed large pleural effusion.   Bedside echo showed pericardial fluid as well.  Pt was transferred out for pericardial window.   Be careful with signouts.  Re-eval the patients if things don’t fit or they change.  Be careful with cancer patients and consider all possible complications of cancer.  Pleuritic chest pain could be PE but alternatively it can be pleural effusion or pericardial effusion.

 

Case 10.   Pt with ludwig’s angina.   Michelle felt pt should be intubated. Anesthesia/ENT/Pulmonology consultants all evaluated the patient in ED and recommended against intubation.   Pt was treated with IV decadron and IV antibiotics.   Pt remained stable over the next 3 days and was never tubed.   Have a clear plan in your head for the difficult airway.   Get help from consultants but also remember that you are the ED patient’s doctor and act on your instincts.   Christine comment:  Most of the time infectious processes in the neck and submandibular area in which you are worried about the airway don’t obstruct.  You can reasonably watch them with aggressive medical management.

 

Kettaneh/Herrmann      STEMI     Conference

 

Case 1.   67 yo male with SOB     EKG  shows Sinus tach with RBBB.    Pt had a seizure and monitor shows V-Fib.   Pt defibrillated and gets ROSC.    Pt had hx of CAD and COPD and was noted to be wheezing s/p defibrillation.   Repeat EKG does not show STEMI.    1hr later patient arrests again.  Defibrillated and gets ROSC.  Pt is alert but is electively intubated.   Pt not given TH because he is alert and communicative.  Repeat EKG shows anterior STEMI.   Harwood comment:  This patient should go to the Cath Lab.  There is evidence that patients with V-Fib arrest with or without  STEMI criteria have improved outcomes if they go to the cath lab.  Even if there is no STEMI,  post arrest patients do better if they go to the cath lab.  It is not known why (more aggressive supportive care including balloon pumps?). 

Recent data supports taking all post-arrest patients to the Cath Lab whether or not they have a STEMI.  Early cath following arrest has higher survival.  Absence of STEMI on EKG following arrest does not exclude coronary occlusion. Elise comment: The EP has to advocate for their patients and get them to the Cath Lab even if they don’t have STEMI post-arrest.

There was a discussion of the high mortality rate of post arrest patients who go to the Cath Lab and the problem of public reporting of Cath Lab data.   There was discussion of  selecting patients for cath post-arrest based on co-morbidities.

Visiting cardiologist at conference mentioned that patients with out of hospital arrest and prolonged down time,  poor or no CPR have a 100% mortality in our Hospital.   So they will not take those patients to the Cath Lab.  Bottom line: Better outcomes are expected for patients with shorter down time, immediate high-quality CPR,   and less co-morbidities.

 

Case 2.   53yo male with chest pain.  Pre-Hospital EKG shows  antero-lat ST elevation and high lateral minimal ST depression.    EKG in ED is less worrisome and repeat EKG’s showed no evolution.  Pt was managed medically for first 2 hours.  Patient had mild elevation of troponin.   Was cathed next day. 

 

There was a discussion of STEMI criteria and specifically LBBB.  Elise comment: If a patient has an AMI causing a new LBBB they should look like crap.   If enough myocardium is damaged to cause a LBBB the patient should have hypotension or signs of shock or arrythmias.   Atypical chest pain or mild chest pain with LBBB is not a STEMI based on the 2013 AHA guidelines.  Visiting cardiologist agreed.

 

Harwood comment:  If you are working in a Community ED and you can transfer a STEMI patient to a Referral Center and they can be on the table with balloon up within two hours, don’t start TPA, transfer them.  This is the new 2013 AHA recommendation.

 

Permar     Electrical Injuries

 

Low Electrical Resistace : nerves, muscle, blood, wet skin

High Electrical Resistance: bone, tendon, fat, dry skin

 

Alternating current can cause tetany.  Because our forearm flexor muscles are stronger than the extensor muscles the victim can’t let go of the electrical source due to the electrical current.

 

Cutaneous burns are the most common electrical injury

 

AC is more likely to cause V-Fib arrest

DC is more likely to cause Asystolic arrest

 

Respiratory arrest can be due to thoracic tetany, CNS injury, or cardiac arrest.

 

Check for rhabdomyolysis and compartment syndrome in patients with electrical burns.   Relatively minor appearing injuries can have significant muscle injury underlying.

.

Get EKG on all patients.    Consider CPK and other lab testing based on severity of injury, loc, neuro symptoms,  cardiac symptoms, pain and signs of burns. 

 

Kids can get cord bite injuries.  These can develop oral commissure  burns that can have delayed labial artery bleeding when eschar falls off.    Discussion among attendings:  no one has seen severe bleeding from such an injury. 

 

Lightning can cause cataracts and TM rupture 

Lichtenburg figures

 

 Keraunoparalysis is a temporary paralysis specific to lightning injuries that is characterized by blue, mottled, and pulseless extremities (lower more commonly than upper). These findings are felt to be secondary to vascular spasm and often resolve within hours but can be permanent (Up to Date,   Environmental Electrical Injuries,  Pinto DS, et al)

 

Tasers injuries are rapid, low amp DC shocks.    No need for EKG or other work up related to the shock.    Only deaths were related to concomitant PCP or cocaine use.   Patients can be safely dc’d after dart removal and evaluation for other injuries.  There can be secondary traumatic injuries due to falling after being tased.

 

Carlson     Mock Oral Board Wrap Up

 

Case 1.  HSP   Perform skin exam, recognize HSP, control pain,  advise surgeon against surgery.   Be sure to undress the patient.  HSP is a small vessel vasculitis.  Immune mediated.  Classic rash is purpuric on lower extremities/buttocks.  Patients can have collickly abdominal pain or GI bleeding.  Hematuria and arthritis are common.

 

Case 2. SCFE    Identify slipped capital femoral epiphysis.  Manage pain,  make patient non-weight bearing,  consult ortho for surgery.  Main complication is avascular necrosis. 

 

SCFE on Patient's left side

Case 3.  Sickle Cell disease  in a pediatric patient with pain crisis and cholecystitis/cholangitis.   Need to manage pain crisis,  give IV zosyn or unasyn,  consult surgery for Cholecystectomy.

 

Case 4.  Salicylate poisoning    Pt needs to be treated with alkalinization initially but eventually requires dialysis due to worsening CHF.   Replace potassium because alkalinization will be ineffective if patient is hypokalemic.   Recognize signs of volume overload from bicarb drip and move to dialysis.  Realize that intubating an  ASA toxic patient can be dangerous because it is difficult to keep up with their minute ventilation requirements and they become more acidotic.   Indications for dialysis is basically they are getting sicker despite alkalinization

 

Case 5.  Acute angle closure glaucoma   Must get visual acuity.  Measure IOP.  Treat with Topical miotics (alpha agonist, beta blocker, pilocarpine)  Carbonic anhydrase inhibitor (Acetazlamide) and/or osmotic agent (mannitol).   Consult optho emergently.  Treat pain and nausea.  Girzadas mnemonics:   E=m2C  (miotics, mannitol, carbonic anhydrase inhibitor)   TAP (timolol/apraclonicdine/pilocarpine)  the cornea with the Tonopen

      

Case 6.  Thyrotoxicosis that developed secondary to untreated hyperthyroidism and pneumonia.   Treat with PTU (block synthesis), SSKI (block release), propranolol (block peripheral effects), decadron (block T4 conversion).   Can substitute gastrograffen for SSKI.  Give antibiotics for pneumonia.  Dexamethasone is the only med that improves long term outcome in thyrotoxicosis.   Avoid ASA, amiodarone, and IV contrast.  Harwood comment:  Propranolol is the beta-blocker that best crosses the blood/brain barrier and improves CNS symptoms.

 

Case 7. WPW with AFIB and aberrant conduction     Attempt procainamide initially  and when patient worsens cardiovert.  Admit for ablation.  Dangerous actions for wide complex, irregular rhythm is adenosine, beta-blocker, calcium channel blockers, digoxin.  Anything that slows AV conduction will increase conduction through the bypass tract and worsen clinic condition.

 

 

 

Case 8.  3rd degree heart block due to Lyme disease with carditis     Patient needs  attempt at transcutaneous pacing and when that fails proceed to transvenous pacing.   Patient required IV antibiotics (Rocephin,  Cefotaxme, Pen G, IV doxy should work as well if patient has severe PCN allergy)  PO doxycycline is not adequate for Lyme Carditis.  Be sure to provide sedation/analgesia prior to transcutaneous pacing.   Heart block will resolve with IV antibiotics.    3rd degree block has a wide QRS and there is no consistant  PR relationship.  There is AV dissociation.

 

 

Patel         Study Guide     Wound Management

 

I am sorry but I missed the majority of this excellent presentation.

 

Storage of amputated digit:  Wrap body part in saline moistened gauze.   Place body part & guaze in plastic bag and then place plastic bag on ice. 

 

Wound prep prior to laceration repair: do not shave surrounding hair.  Ok to clip hair or mat hair down away from wound with antibiotic ointment. 

 

Tissue adhesive has similar cosmetic outcome to sutures. 

Saddle nose deformity is the complication of untreated nasal septal hematoma.

 

Christine comment:  This is bread and butter emergency medicine.  You must read this chapter! 

 

 

 

Conference Notes 3-5-2014

Happy EM Residents Day!   The RLT and Faculty  just want to give another Huge SHOUT OUT to our Residents,  THE  Greatest Residents On the Planet!  THANK YOU!!!   

It was another very educational Conference Day, Hope you enjoy the notes.

Frazer/Paquette     Oral Boards

 Case 1  33 yo pregnant female (27 weeks) presented with abdominal pain.   Labs showed a leukocytosis, anemia,  low platelets and dohle bodies.   LFT’s were jacked up.   D-dimer was 23.   Diagnosis was HELLP syndrome.    Fetal Heart tones were 80.   Patient placed in left lateral decubitus position and OB called to take patient to OR for delivery.  Platelets were given.    HELLP  more common in multigravidas.  Pregnant patients with RUQ pain/tenderness and  nausea, vomiting, and malaise being diagnosed with a  viral illness, hepatitis, or GB disease is a common pitfall that has resulted in maternal death or severe morbidity

Case 2  25 yo male presented with left forearm pain following a fight.  Pt was struck in the forearm with a pipe.  X-rays are negative for fracture.  Pt has marked pain and muscle compartment of mobile wad is hard to palpation. Compartment pressure was markedly elevated. Diagnosis was compartment syndrome.    3 compartments in the forearm: dorsal, volar, mobile wad.  Treatment was emergent fasciotomy. 

  •  Acute Compartment Syndrome  delta pressure = diastolic blood pressure ‒ measured compartment pressure
  • Acute Compartment Syndrome delta pressure < 30 mmHg indicates need for fasciotomy.   
  • (Up to Date   Compartment Syndrome)

 Case 3  60 yo female presented with acute psychosis.  Only lab abnormalities were elevated BUN and hypercalcemia (level=15).   CT head was negative.    IV fluids, calcitonin, and a bisphosphonate were given. 

Severe hypercalcemia — Patients with calcium >14 mg/dL require  aggressive therapy. The acute therapy of such patients consists of a three-pronged approach

  • Volume expansion with isotonic saline at an initial rate of 200 to 300 mL/hour that is then adjusted to maintain the urine output at 100 to 150 mL/hr
    Loop diuretic therapy is not recommended because of potential complications and the availability of drugs that inhibit bone resorption, which is primarily responsible for the hypercalcemia.
  • Administration of salmon calcitonin (4 international units/kg)
  • Administration of a bisphosphonate (zoledronic acid)
  • Saline and calcitonin take effect in 12-48 hours.  Zoledronic acid takes effect in 2-4 days
  • (Up to Date   Management of hypercalcemia)

Steele     ABEM Visiting Lecturer

 30, 000 Active ABEM Diplomats

Emergency Medicine  is the 23rd Recognized specialty

ACEP started in 1976

ABEM became a primary board in 1989

Mission of ABEM: To ensure the highest standards in the specialty of EM.   2nd mission is to enhance the value of board certification for emergency physicians

All questions on Board Tests have to come from the Model of EM Practice (on ABEM website)

Qualifying test is on a single day and lasts 6.5 hours.  You have to score 75 to pass.

Oral exam is also a single day test that takes 4 hours

There is a video on the ABEM website of how the Oral Board Exam is administered.

2013 Application fee =420.     Qualifying exam =$960     Oral Exam=$1,225

Time limit from end of training until you pass the qualifying exam is 5 years.  After you pass the qualifying exam you have 5 years to pass the Oral Exam.

Kelly Williamson comment:  Hospitals you work for will not be this lenient.  If you are not board certified in the first year or two, they will fire you. 

 

Steele     ABEM Visiting Lecturer   Career Satisfaction in EM

87% of EP’s are satisfied.   65% are highly satisfied.   1/3 felt burnout was a significant problem.   Diversity of activities that are part of an EP’s job (clinical, academic, administrative) correlated highly with career satisfaction.

 Variables correlated with Career satisfaction:  Leadership roles,  job feels personally rewarding, job security and fair compensation, involvement with teaching/consulting/political activity,  supportive environment. 

 Variables correlated with low Career Satisfaction: not enough personal time, high census ED,  longer length of shifts, night shifts, problematic colleagues.

 Factors important to women in EM:  Recognition at work, career advancement opportunities, schedule flexibility, fair compensation, academic practice.

 Sources of stress: erratic schedule, unreasonable patient requests, fear of making mistakes, threat of malpractice, problems with nursing staff.

 REM sleep usualy occurs toward the end of sleep cycle.    So if you are awoken early, you loose REM sleep.   

 Night owls tend to tolerate night shifts better than Larks (morning people).    Flexible sleepers tend to tolerate shift work better than people who need optimal conditions to sleep.

 Shift work tends to take a toll on psychiatric health,  relationships, and can increase the risk of substance abuse.

 Working a series of 5 night shifts in a row results in a substantial decrease in cognitive performance.

 Naps can improve cognitive function and reduce performance lapses.

 Institute of medicine recommends against 12 hour shifts.

 Gore    Case  Follow Up

 36 yo female, 1 week post partum,  presents with headache and neck stiffness.   Pt also has chest pain and SOB.   Vital signs are normal   Physical exam is normal except for heart murmur. Initial thinking was to focus on the headache as possible post-LP headache, meningitis or SAH.   However being diligent emergency docs other tests were performed to evaluate a broad DDX.  Troponin was neg.   D-dimer markedly elevated.   CXR and CT head were normal.   

 CT chest for PE was done based on the elevated D-Dimer  and it demonstrated an intimal flap in the ascending aorta.  TEE showed aortic regurgitation and dissection.

 Pregnancy is a vascular stress test.      This pregnant patient had a broad differential diagnosis.  Clues to dissection: Chest pain and….. headache,  heart murmur,  elevated d-dimer.   Anytime a patient has chest pain and some other complaint, especially neuro or vascular symptoms, think dissection.   

D-dimer in dissection may (or may not) help risk stratify patients with Chest pain for aortic dissection.   Low levels lower risk.  High levels increase risk.  This is still a controversial test in this setting.  There are no prospective RCT’s evaluating it’s effectiveness.  Most of the studies are retrospective case series.   It worked here though!

Manage blood pressure and aortic pulse pressure with nicardipine and esmolol.  Ascending dissections require surgery.   Pain management is also critical. 

Harwood comment:  In the post-partum period management of aortic dissection is the same as in the non-pregnant patient .     You should make note that the ED physician had to be perseverant  with diagnostic testing and consultations until the diagnosis was rock solid and the patient went to the OR.    Elise comment: A take home point is that courage of conviction is critical for the emergency physician.  It was important in this case to keep pressing on for the correct diagnosis.

Harwood comment:  There have been a number or cases recently in the press of vascular catatrophies during the peri-partum period.

 

DenOuden    Case Follow Up

 53 yo male with SOB, 25 lb weight gain, leg swelling,  polydipsia/polyuria.  BP at clinic was 235/135.

No PMH

Exam shows: 3+ edema bilat, acne and darker skin.   Labs show low K and elevated bicarb.  VBG shows metabolic acidosis.  BNP is minimally elevated.   Pt has leukocytosis.   ECHO shows nl contractility and no pericardial fluid.  EKG demonstrates sinus rhythm with LVH.  Right sided perihilar mass noted on CXR.

 Cushing’s Syndrome is the diagnosis.   Mnemonic for  symptoms/signs of Cushings:

CUSHINGOID      C=cataracts  U= Urine potassium is high and serum potassium is low     S= skin changes with hyperpigmentation, easy bruising and striae  H=hypertension     I=Increase risk of infections    N=Neuropsychiatric  symptoms (anxiety, paranoia)   G=Glucose intolerance    O=Osteonecrosis of the femoral head/Osteoporosis      I=Increased androgens in women, Increased fat in the supra clavicular fossa.       D=DVT risk.  

 Adipose in the supraclavicular fossae that obscures the clavicles is a relatively specific sign of Cushing's syndrome

 

Supraclavicular Adipose

 

 

5% of Cushing’s Cases are due to paraneoplastic syndromes.

The diagnosis in this case was a small cell lung cancer with a paraneoplastic syndrome causing Cushing’s syndrome.

 

Carlson   Oral Boards  Primer

 Practice your opening statement.  “ When I walk into the room what do I see, smell, and hear?”  

Talk to the patient initially for a brief time then decide whether to start evaluating the ABC’s.  If they can’t adequately respond to your initial question, get moving on the ABC’s

Give your nurse some initial orders to start off, “undress patient, IV/O2/monitor/pulse ox

 Consider all sources for history including EMS, family, friends, and patient’s primary doc.  Always ask for medications and allergies.

Do  a systematic physical exam on all patients.    The examiner should move you along through normal exam areas.   Be sure to ask for skin exam.  If the examiner asks you “what are you looking for?”  That is commonly a clue that the exam area being asked about has a significant finding. 

 Re-assess the patient regularly.  Treat pain right away.

 Thinking  out loud can be very helpful if you know what is going on.   If you don’t know what is going on be careful about thinking out loud so that you are not wasting time or letting the examiner know you don’t know what is going on. 

 You should know ACLS med doses.   You usually can use references/consultants for non-ACLS doses.     Don’t give a specific dose if you don’t know it.

If you get lost, review the positives you have noted on H&P and diagnostic testing and work from there.

 For Boards,  Know ACLS algorithms,  indications/contraindications for thrombolytics, antidotes, and procedural techniques. 

 

Feb 12, 2014- CKulstad subbing for Dr. Girazadas who is attending AAEM

8-9 Procedural sedation Study guide-  Lovell

Minimal sedation = anxiolysis. Spontaneous breathing, airway unaffected.

Moderate sedation- purposeful response to verbal or light tactile stimuli. Standard examples- midazolam/fentanyl

Deep sedation- what actually happens during procedural sedation most of the time. Responds only to painful or repeated verbal stimuli. Airway reflexes may be lost, respiratory effort may be inadequate.

General anesthesia- all reflexes lost. Must support airway and possibly CV system

Dissociate sedation- trancelike, cataleptic state. Airway reflexes maintained

Sedation is a continuum- be prepared to treat someone on a stage deeper than you intend.

NPO-ACMC: No solids 8 hrs prior, no clear liquids 2 hrs before. Anesthesia national guidelines say 2 hrs npo for clear liquids, 4 hrs for breast milk, 6 hrs for solids. Harwood adds these guidelines originally derived from c-section data on term pregnant women who have very high vomiting risks

This is out-dated. ACEP guidelines say fasting rules not supported by evidence.

ASA classifications- procedural sedation generally only for patients in category 1-3. Add “E” to the category for “emergency” and you’re saying you have to do procedure (pulseless limb s/p dislocation)

1-      Healthy

2-      Mild systemic disease that is well controlled

3-      Severe systemic disease- eg symptomatic wheezing in COPD

4-      Life-threatening illness

5-      Dying patient

6-      Brain death

Tips for safe sedation- use monitoring (ECG for hx of cardiac patients). You’re in charge of everything. If you use Demerol, it can cause CNS excitation (seizures). Fentanyl can cause respiratory depression, esp in elderly. Generally better to give meds slowly and titrate doses.

Dosing

Midazolam (Versed) 1mg (0.025 mg/kg)

Fentanyl (Sublimaze) 25-50 mcg (0.5-1 mcg/kg)

Flumazenil for benzodiazepines only 0.1-0.2 mg

Narcan for opioids only- suggested dosing- dilute 0.4 mg in 10 ml normal saline, then 1 ml/dose

-5 ways to decrease pain of local anesthesia- add bicarb, warm it, inject slowly, use small needle, distract patient, inject through wound margins

-Eutectic mixture of local anesthetic is what EMLA stands for. Other fast acting topicals- LET (lidocaine- epinephrine-tetracaine) for not-intact skin, ELA-Max-liposomal lidocaine for intact skin

-Side effects of ketamine- rare laryngospasm. More likely with suctioning or with lots of secretions. Usually able to bag someone through it. Can use in kids 3 months or older. OK to use in head injury but not in hydrocephalus or known increased ICP. Additional benzos not recommended for kids, is for adults.

-Treatment with opioids in ED is not the same for all racial/ethnic groups (JAMA 2008). Be aware of your biases.

-Can treat benign headache with injection of local anesthetic (bupivacaine) in paraspinous muscles in lower cervical spine. See EMRAP or Youtube for more details (Dr. Mellick).

-Local anesthetic allergy: 2 classes amides (lidocaine, bupivacaine, prilocaine- all have ”i” before “caine”) esters (procaine, benzocaine, tetracaine). If allergic to both, can inject diphenhydramine as local

-Toxicity of lidocaine- CNS (seizure, coma) then CV (dysrhythmias, myocardial depression). Before that get symptoms that sound like anxiety (perioral numbness, not feeling right). Treat with benzos then amiodarone. Short lived toxicity which Andrea has never seen. Intralipid for bupivacaine overdose.

-Nitrous oxide needs to have a well-ventilated room, can go into gas filled cavities so avoid in ptx, sbo, balloon-tipped catheter. Altered patients should not get NO as patients control their dose.

-Discharge requirements- ambulate, responsible person to watch (no driving for 24 hours), normal vital signs. See modified Aldrete score for more details

-Max dose local anesthetics: lidocaine 4 mg/kg plain 7 mg/kg with epi. Bupivacine 3 mg/kg plain, 5 mg/kg with epi

 

9-930: Geriatrics in ED – Beckemeyer

Geriatric population is increasing- 20% by 2039, and 25% in 2050. And >65 fill out surveys.

Every ED visit  at age >65 is a sentinel event for further decline so transitions in care critical. Social work, home visits, prompt PMD visit, action plan for decompensation.

4/2014 roll out date for new geriatric ED experience. Goal- age >65 is no wait for all complaints. Go to “senior care area” in former GC front area. Have specific assessment protocols (ADL/med questions, fall risk, social support), and specially trained techs.

Will also have specific protocols so care can be started prior to MD eval for common geriatric problems.

Discharge packet will include senior specific support info (eg meals on wheels).

Will use grants for inexpensive changes- eg clocks with large numbers, magnifying glasses, bedside commodes, more comfortable carts, more pillows and blankets available, possible carry Ensure in ED.

Try to speak more slowly and loudly- face patient at eye level. Elderly patients often overwhelmed/scared in ED.

930-10: Safety lecture- Cash

Had unscheduled downtime early Jan- Firstnet, allegra, pacs, phones all down. Code triage called, went on bypass. Concern for safety issues.

Issues- unclear if orders when through prior to downtime. Delay in identifying which patients needed to be seen. Hard to track workup for find results. Hard to identify acuity, when patient for which team. Patient lost to system. No one familiar with paper system.

Scheduled downtime for computer maintenance- extra staff and materials ready. There are few downtime computers throughout ED- very basic list of patients. One for main room adults, one for fasttrack and peds.

Use whiteboards and paper packets. Whiteboards will list patient by color (red, blue, and black for gold team). Make sure papers have patient stickers. Lab results are faxed to ED and placed in physical chart- you have to keep checking.

To discharge patients find a downtime computer and hit depart. Discharge instructions under “patient ed” which pulls up your usual discharge instructions. Can also use uptodate patient instructions.

10-1030: Advanced DKA

Brian Febbo is a second year resident.

DKA defined by serum or urine ketones, glc usually >250, anion gap acidosis

Euglycemic DKA (below 250) exists. It is not equivalent to mild DKA.  Associated with continued insulin use as DKA develops, pregnancy, and starvation.

If a patient has low albumin, the anion gap may be falsely low. Correct by adding 2.4(4.4 – [albumin]).

Mixed acid-base disorders are common- vomiting can falsely normalized pH.

Serum ketones would be very helpful but are not available in ED at ACMC. Urine ketones have other causes, so less specific.

HHS- older, sicker, more dehydrated. Will need much more fluids. Give IVF alone for first couple of hours as they will significantly drop glucose and potassium. These patients need extensive workup.

Look for precipitating factors. Usually infection, lack of insulin, or other critical illness (medications, pancreatitis, MI, PE, other endocrine abnormality).

But non-specific lab abnormalities are very common- especially leukocytosis and lactate- so difficult to diagnosis.

Management- Fluids early and aggressively. Use isotonic fluids, will need to change additives throughout.

Insulin- 0.1 U/kg/hr or 0.14 U/kg/hr gtt. If you want to give a bolus, it is also 0.1 U/kg but utility is questionable.

SubQ  insulin used to good effect in mild or moderate, stable DKA patients. They need an IV or IM bolus dose.

Remember about pseudo hyponatemia- use corrected to calculate AG

Hyperchoremic acidosis common with large volume IVF resuscitation so consider using LR

Must check potassium before starting insulin- can trigger malignant arrhythmia if it was low and you give insulin.

DKA in ESRD- total body water is near normal, most in extracellular space. Treat with insulin gtt alone.

Pediatric DKA- no evidence that aggressive IVF repletion causes cerebral edema but standard practice in US is to replete fluids over 48 hours.

1030-1230 Sedation small groups

 

Conference Notes 2-5-2014

 

Gore/Konicki        Oral Boards

Case 1. 30yo female with sore throat.  Patient is 5 days post partum.  No fever.  On exam pt has an enlarged uvula.  Diagnosis was idiopathic angioedema.  Treatment is H1 blocker, H2 blocker, steroids.  Consider  EPI if necessary.    Lovell/Motzny/Girzadas comments: This is uvular hydrops or Quinke’s disease.  Treat with steroids, histamine blockers, racemic epi nebulizer treatment, cold fluids or ice/popsicles at home.  Check for strep.   Don’t overdo the work up.  No need for CT or xrays in these patients.  

                                                                                                                                                                  Quincke's Disease/Edema

 

Case 2. 57yo male with marked jaundice and abdominal pain.  Light colored stools/dark urine.  No fever.  Pt has history of ulcerative colitis and is on an azole drug.    Diagnosis was cholestasis.  Can be due to impaired processing or obstruction.  Pt had primary sclerosing cholangitis.  Primary sclerosing cholangitis is associated with inflammatory bowel disease.   Image the liver and GB with U/S or CT. Consult GI.   Main thing is to rule out acute cholangitis.  

Case 3. 40 yo male with crush injury to right hand.   Pt had open finger fracture.  Critical actions: digital block for pain management. Opioid pain medication,  Neuro-vascular exam.  Tdap, prophylactic antibiotic,  irrigate wound, splint, cover open wound.   Elise comment: Be specific and clear what you want the consultant to do. 

 

Febbo      Pearls and Pitfalls in the Emergent management of HTN

Treat asymptomatic HTN cautiously or not at all.  You want to avoid overshoot hypotension and possibly cause a stroke or MI.

Hypertensive Emergencies have end organ dysfunction.

Hypertensive urgencies and malignant hypertension are terms that have fallen out of favor.   Papilledema was thought to be a marker of malignant hypertension but this has more recently been found to be unrelated to end organ dysfunction from HTN.

BP stabilizes in a patient at about an hour into an ED visit.  This may be the optimal time to get a blood pressure reading.

End Organ Damage: Encephalopathy, pulmonary edema, acute kidney injury, preeclampsia,  aortic dissection, acute coronary syndrome, ICH, CVA.

What testing is required: Screening tests (Labs, CXR, EKG, CT) don’t help.  You should do targeted testing for specific symptoms or signs.  Normal urine dip predicts a normal creatinine.   ACEP Guidelines recommend against routine screening.   In selected patients, Cr screening may be indicated. 

There is no evidence to suggest any benefit of lowering BP acutely in the ED.  There are many case reports  that describe iatrogenic negative effects due to acute overtreatment of assymptomatic HTN.

Elise/E Kulstad comment: We have both seen patients who have developed stroke symptoms stemming from over-zealous BP lowering.    Motzny comment: Most patients have some symptoms like headache or weakness.  We have to decide on these patients whether to work them up.  There was then a discussion among the attendings about who needs a CT head for headache.  Most agreed that the higher the BP, the older the patient, and possibly the more concerning the headache symptoms the more likely they would be to do a head ct.   This was a grey zone for everyone though. 

Hypertensive Emergencies: Encephalopathy, pulmonary edema, aortic dissection, acute kidney injury, preeclampsia.     Most patients with severe hypertension are volume depleted.  Volume depletion can make patients very sensitive to BP treatment and make them more likely to become hypotensive.  IV fluids can be used to treat a patient if they become hypotensive due to BP management.  The one exception to this is pulmonary edema. 

 Go-to med to most hypertensive emergencies is Nicardipine.  Start at 2.5mg/hr and titrate by 2.5 mg/hr Q 15 min.

Labetalol is second choice.  Dosing is 20mg q10 minutes up to 300mg. Avoid this drug in patients with a history of asthma/bronchospasm/COPD

NTG has a niche use for acute pulmonary edema.

Esmolol is good for rate control but not indicated for BP control.  Can be used as combined therapy  with nicardipine to reduce wall stress in aortic dissection.

Fenoldopam:  Peripheral dopamine agonist that maintains renal perfusion.  Dosing is 0.1mg/kg/min to start.   It is pretty expensive.

Agents to avoid: Hydralazine, clonidine, ace-inhibitors, nitroprusicsside. 

Aortic dissection: Pain meds, esmolol, nicardipine

Preeclampsia: Magnesium +/- hydralazine or nicardipine.  Hydralazine is falling out of favor.

Ischemic stroke:  If thrombolytics are being given get bp less than 180/110.  If no lytic being given, don’t treat BP if less than 220/120.

ICH: Nicardipine

Sympathomimetics (cocaine, crystal meth): Benzos

Pheochromocytoma:  Use phentolamine

 

Lambert      Vascular Access Workshop  

High frequency linear probe is the optimal choice for vascualar access procedures.   It gives higher resolution with less depth penetration.   That performance profile is usually best for visualizing vessels.  You want optimal resolution of the vessel usually at a shallow depth beneath skin.  

IJ Placement: When the operator is working from the head of the bed to place a line, keep the indicator of the probe pointed to the operator’s left.   Visualize the IJ in the transverse orientation and center it on the screen.  Be sure the vessel is compressible to rule out a thrombus.  Puncture skin at a 45 degree angle.  Your puncture point should be the same distance from the probe as the depth of the vessel on the screen.   A trick to this procedure is identifying where the tip of the needle is on U/S.  As the needle tip tents the vessel wall, you have to jab the needle through the vessel wall.

Peripheral IV Placement: Look for a shallow, large vein in the medial aspect of the upper arm.  The basilic vein is usually most superficial.  Make sure it is thin walled and compressible and go for it.  Elise comment: We are limited by the catheters we have.   Mike response: Yes, you need a longer peripheral catheter to do this procedure.  Usually need a 2cm length catheter or longer.

U/S for DVT:   You just need to scan the proximal and distal segments of the femoral vein.  Isolated mid-femoral clots are very rare.    In the popliteal view, the popliteal vein is closest to the skin. (POP on TOP)   In the femoral view, the vein is deep to the artery.    The femoral/popliteal vein should be completely compressible.  If not, it is either the artery or a DVT in the vein.  Clot is not always echogenic, so  a clot may be present if the vessel is not compressible but you don’t see echogenic material in the vessel. Also, you can see baker’s cysts in the popliteal fossa.   The primary criterion for a DVT is non-compressibility of the vessel.

 

"Pop on Top"

 

The Proximal Femoral Vein tends to lie deep to the Artery in Contrast with the Popliteal Vein 

 

Hands-On  Vascular Access Workshop

 

 

 

Conference Notes 1-22-12014

Airway Day  

Much Thanks to Drs. Bolton and Tekwani and all the faculty for this outstanding Airway Workshop!!!

Mistry        Approach to the Airway

LEMON= Look,  Evaluate,  Mallampati, Obstruction,  Neck Mobility.    In the Evaluate section think 3-3-2:  Can you fit three fingers in the mouth opening,  three fingers  from the chin to the hyoid, and two fingers from the notch of the thyroid cartilage to the hyoid bone?  If you can’t do those three things that portends a difficult airway.

 

Your ability to keep calm during an airway emergency affects the demeanor and clinical capacity of the entire team

Build up an oxygen reservoir  with 3 minutes of tidal volume breathing with high FIO2.  Use a non-rebreather mask with the FIO2 as high as possible.   8 Vital capacity breaths will maximize airway/pulmonary  O2 concentration and minimize CO2 concentration.

Bag Valve Mask Ventilation can be life-saving.

Laryngeal Manipulation:  Intubating physician can move the larynx with right hand  to optimize view and then have an assistant hold the larynx in that position.   Alternatively the assistant can perform the BURP maneuver  Backward/Upward/Rightward/Pressure.

Don’t forget to use  the PEEP valve on the ambu-bag when bagging is difficult.

Nasal cannula in the nose with the oxygen flow as high as possible during intubation can increase the oxygen reservoir and buy you some extra time during intubation.  The patient needs to be sedated during this technique as it is uncomfortable for the patient.

Don’t lay the  severely dyspneic patient down until necessary.  The sitting position can optimize their breathing.

Intubating the Obese Patient: Key is  RAMP positioning.  You need to elevate/position the external auditory meatus in line with sternal notch.  You want the patient’s face looking up at the ceiling.  You can use a combination of elevating the head of the cart and stacked linens to get the head positioned with the face looking up and the external auditory meatus parallel with the sternal notch.

 

RAMP positioning

Sedated Laryngoscopy can be useful for the difficult airway.  Ketamine sedation or Ketafol sedation in conjunction with good topical anesthesia of the throat and larynx is a useful approach if you have the time .  Once you have the patient tubed you can then fully sedate and paralyze them.  

There are many airway devices but, your knowledge and planning is the best tool you have to manage the difficult airway.

Lovell                 Airway Devices 

Supraglottic devices such as an LMA can be an excellent bridge device for the difficult airway until you can get the patient intubated.

Elise dicussed  numerous different airway devices.

In the patient who is unresponsive and won’t bite you, you can try digital intubation.  You use your non dominant hand to feel the glottis and lift the epiglottis.  With your dominant hand you pass the tube between your fingers and thru the cords.

Nasotracheal intubation: The patient has to be spontaneously breathing.  You have to time the passage of the ET-tube to the patient’s inspiratory effort.  

Gum Elastic Bougie should be your first “go-to” back up device.  There are no contraindications to it’s use.

There was a discussion concerning the issue of whether video laryngoscopy will render direct laryngoscopy completely obsolete.   Harwood and Elise felt that eventually it would make direct laryngoscopy obsolete  although that is not the case currently.  Old school Girzadas felt that direct laryngoscopy would always be a needed skill for the emergency physician.

Video laryngoscopy devices have been designed with increasing sophistication.

LMA is the bridge technique to temporize the airway until you can get the patient intubated. It is a blind technique that can be done in a few seconds in the crashing patient.  It also can be  the initial airway in the cardiac arrest patient.   An intubating LMA can be used  to get a definitive airway in the patient.  

Cricothyrotomy is the procedure you have to be able to do as a last ditch technique.   Biggest error is waiting too long.  An assistant should be preparing for cricothyrotomy while you are still trying to intubate the patient.   Can’t do cricothyrotomy on patients age 8 or below.  Girzadas comment: You are usually performing this technique on obese patients.   Due to adipose tissue and blood this is, in effect,  a blind technique.  You have to be able to feel with your fingers where you are going.   Lovell and Girzadas comment:  A bougie placed through the incision in the cricothyroid membrane will greatly help you keep control of the airway and place the shiley.

Elise discussed approaches to pictured  specific difficult airways.

Sola comment: If there is a ton of blood in the airway and you can’t identify the cords, mild chest compressions can force some air through the airway and cause bubbling by the cords.  Aim for the bubbles to tube the patient.

Advanced Airway Skills lab

 

 

 

 

 

Conference Notes 1-15-2014

Bolton         Ethics for the Altered Patient

Deciding whether a patient is competent  for decision making is a legal issue.  It is decided by a court.  ED docs for the most part are not involved in competency decisions. 

A patient’s capacity for decision making is a clinical decision.  Determining a patient’s capacity for decision making is a common task for ER docs.    A mnemonic  for to guide the emergency physician  is CURVES: The patient needs to be able to  Choose and Communicate, Understand, Reason, make Value judgments  appropriately,   if an Emergency situation exists then you need to act without his consent, If a  Surrogate is available, they can consent for the patient.

Molly presented a case of an etoh intoxicated patient with a severe head injury.   The patient lacked the ability to choose/communicate/understand and reason.    He had no surrogate present.  There was an emergency condition present.   Appropriate action would be to sedate the patient and get CT of head.  There was some disagreement in the audience about whether the physician could also suture the laceration prior to the patient becoming sober and giving consent.  Majority felt repairing the laceration while patient was sedated was ethical.

Patients have a right to have untreated chronic psychiatric disorders.  However, they can’t refuse treatment for psychiatric disorders  and be homicidal or suicidal or not take care of themselves.   But if patients have a chronic stable psychiatric illness and they are not suicidal/homicidal/unable to care for themselves they can have the ability to refuse treatment or consent to treatment.   Schizophrenics don’t necessarily diminished capacity for decision making.   Acutely psychotic patients lack capacity due to lack of understanding and inability to reason effectively.

Suicidal patients are difficult.   They may seem to have capacity.   Molly presented the case of a depressed  45 yo male who had lost his job, house, and wife.  The patient ended up in the ED and told the physician he would go home and shoot himself.    Molly said that the patient’s depression impacted on the value part of the CURVES mnemonic.   His desire to kill himself is not consistent with his pre-depression value system.   From a practical  standpoint, failure to prevent suicide is one of the highest payout malpractice categories for emergency physicians.   If a patient is at risk for suicide the emergency physician has a duty to prevent suicide.

Next case was a 57 yo female with acute aortic dissection who wants to leave hospital AMA.  Pt states she needs to go home to care for her pets.   Discussion covered  issue  that patient doesn’t have capacity based on lack of reasoning ability.   Harwood comment:  If you can get the patient’s concern solved like getting a neighbor to feed the cats or getting the police to pick up a child at school you frequently can get patients to stay for treatment.   Christine comment: I tell the patient that I have the right to keep you here in the ED until you can demonstrate to me that you actually understand  and can rationally explain the choice you are making.   Elise comment: AMA doesn’t protect you.  What protects you is a thoughtful conversation with the patient about their options and risks and carefully documenting that conversation.   That conversation and documentation may provide some protection.   Kelly comment: if patient leaves against your advice give them good discharge instructions to return at any time and give them the best options for home care.   Girzadas comment: Get family members involved as well.  They may be able to get through to the patient.  Also for risk mitigation involving family members can be critical because they are the ones likely to sue if the patient dies.

Collander                            M&M

75yo female with chills and leg shaking.  Pt is on chemotherapy for stage 4 rectal cancer.  She also has polymyalgia rheumatica and is on chronic prednisone.    Rectal temp in ED was 39.2. (recent labs were known and pt was not neutropenic so rectal temp was done)   BP was 81/50 and heart rate is 115. Lactate was not ordered in the initial order set.

Vitals improved slightly with IV fluid 1 Liter bolus.   Pt started on Vanco/Zosyn/Azithro and normal saline at 125ml/hr.   Chemo infusion was ongoing in the ED.

BP dropped to 60’s systolic.  Pt is tachycardic again.   2nd NS bolus started.  Lactate ordered now and comes back  7.7.   Pt was transferred to the ICU.   She recovered and did well.

If a patient has SIRS and a low MAP give 30ml/kg crystalloid IV .     If lactate elevated or pt remains hypotensive after bolus or pt has organ dysfunction then the patient has either severe sepsis or septic shock.  At this point, Early goal directed therapy for sepsis is indicated.   Elise comment: Scvo2 can be obtained by running a blood gass off of the central line.

Lactate is due to anaerobic metabolism . Lactate is a marker of patient status.  If at 6 hours lactate is decreasing, pt’s mortality rate has improved.  If lactate has increased, mortality increases.  .  Blood that sits in lab for a prolonged time can have a falsely elevated lactate.  VBG is most accurate lactate

Elise comment: Respect rigors, it is a marker of bacteremia.    Harwood: This patient was on chronic steroids for PMR.  I would have given stress dose steroids for this patient.    Christine/Elise/Harwood comments:   If  a patient has sepsis fill up the tank with 30ml/kg bolus.  Every patient gets this amount of fluid no matter how old, or how frail, or on dialysis, or CHF.  The only patients who you don’t give this dose of fluid to are patients who are DNI.  If a patient gets volume overloaded you can solve the problem with intubation.  DNI patients on the other hand don’t have this therapeutic option available so you will need to be more cautious with them.

Konicki                     Urologic Emergencies

Urinary retention:  Causes include BPH, constipation, prostate cancer, urethral stricture, uti/prostatitis or medications.   Use a lidocaine urojet to facilitate catheter placement.   Larger Foleys have more stiffness and may be more likely to pass through prostate than smaller foleys.  Standard catheter choice for difficult patients is a coudet catheter.  Lay the patient completely flat.  Pull up the penis to make the urethra as straight as possible. Hold the catheter like a pen.  When you hit resistance ask the patient to try to urinate and this sometimes allows the catheter to pass.   If you are unsuccessful passing a foley using the above techniques, consult Urology to place foley using the obstructed urethra tray/kit.   If you are faced with a significant delay to arrival of urology, you can consider doing an U/S guided suprapbuic tap to drain urine.  This is a safe procedure.

Paraphimosis:  Place penile block.  Apply compression to penis by hand or with compressive ace wrap for 10 minutes to squeeze out edema.   You can also apply sugar, mannitol  or dextrose soaked gauze around penis to act as an osmotic agent to reduce foreskin edema.  Ice also can aid as a vasoconstrictor on the penis.   Once the foreskin edema has been reduced,  grab foreskin to reduce over glans.   Another approach to reduce foreskin edema is to make multiple small punctures in the foreskin with a 25g needle.

 Priapism: Using a non-heparinized 19-gauge butterfly needle, approximately 5 mL of blood should be aspirated to decompress the corpora. Phenylephrine should then be injected into one side of the corpus cavernosum. In adults, phenylephrine should be diluted with normal saline to provide a final concentration of approximately 100 mcg to 500 mcg per mL. One mL intracavernous injections of the freshly diluted phenylephrine solution are administered every three to five minutes until resolution or up until one hour, before deciding whether the treatment will be successful [22]. Lower concentrations and smaller volumes are appropriate for use in children and those with severe cardiovascular disease. It may be necessary to repeat aspiration and phenylephrine injections over several hours to achieve detumescence.  (Up to date)

Testicular torsion: Do a bedside U/S with both testicles in the same view so that you can compare them.  If they both have similar appearance of blood flow, that is a good thing.  If one teste has less flow, you need to be calling Urology emergently.    To detorse the teste,  you can try “opening the book” .  Maletich comment: I reduced a testicle recently.  We initially tried open book technique but pain worsened so we rotated in opposite direction and pt’s pain improved and testicle dropped down into normal position. 

Bedside renal U/S for renal colic:  It helps to lay the patient on the side opposite the pain.  Put the probe in the intercostals space of the lower ribs.   Elise comment: I would do a CT for a patient who had signs of uti, first time renal colic,  suspicion for AAA.   Joan comment: U/S or CT imaging is important also to rule out single kidney.   Use the color flow Doppler on the suspected kidney, if there are large areas of no flow, that is hydronephrosis

Harwood comments: To deflate a foley balloon that won’t go down with aspirating at the inflation port, you can cut the inflation channel and thread a guidewire down the inflation channel.   Paraphimosis can be treated with granulated sugar as an osmotic agent.  Priapism can be treated with withdrawing blood from coropora cavernosum and instilling phenylepherine.

 

Chiefs     Sign Out Culture

Discuss any consults with the admitting physician prior to contacting a consultant. 

Do not sign out a patient who has vascular access issues or if the techs can’t get labs.   Solve these issues before you sign out/go home.

Do the pelvic exam before you sign out the patient.   

Just because your attending leaves early or on time, doesn’t mean you leave at that time.  You may need to stick around until the end of your shift to get things finished up on your patient.

Don’t sign out a patient with whom you don’t have a plan yet.   You need a work up and management algorithm to guide the accepting team.    Elise comment: Just be careful that you are avoiding premature closure on a patient. Harwood comment: There are times when you have to dump a patient.  If you pick up a patient late in the shift and they need a lot of testing you may need to sign that patient out without a clear cut diagnosis/dispo plan.  However, there shouldn’t be more than 1 “dump” per signout.

Update your patients on the plan of management and disposition going forward prior to signing out.  You don’t want to leave the receiving team holding the bag with having to discuss the management plan and dispo with the patient.

 

Anneken                         Fluids and Electrolytes

Na drops by 1.6 for every 100 mg/dl of glucose over 100.

Hypocalcemia results in long QTc.   Hypo k,  hypo MG, and hypo CA all cause prolonged QTc.  Remember HypO=lOng QTC.

In children with diarrhea restart regular diet as soon as possible.  If they are tolerating po there is no need for a BRAT or other form of restricted diet.

Calcium treatment is the fastest modality to treat hyperkalemia.  It acts in 3 minutes or less.  It stabilizes cardiac membranes but does not decrease K.    Insulin/ D50 is the most reliable way to lower K.  Albuterol nebs Q 30 min can be effective but 40% of patients won’t respond.   HCO3 is only effective if patient is academic.   Kayexelate is losing favor and is controversial.   It is still being used on a regular basis. 

Hypertonic saline for hyponatremia is indicated only for coma, seizures, or focal neuro signs.  Also use only in patients with acutely developing hyponatremia such as marathon runners, ecstasy users, or psychogenic polydipsia.   If you give it, no more than 100ml at a time and recheck Na every 2 hours.  Don’t raise sodium level more than 1 per hour.

Osmolar gap= 2NA + bun/2.8 + glucose/18 + etoh/4.6

Normal anion gap acidosis: adrenal insufficiency   mnemonic is USED CAR =    Ureteroenterostomy, Small bowel fistula   Extra CL    Diarrhea   Carbonic Anhydrase inhibitoRs

Treatment for hypercalcemia is normal saline followed by calcitonin and biphosphonates.   Diuretics no longer recommended.  

Acid Base Problems:  ABG analysis

Step 1 decide if the ph is academic or alkalemic

Step 2  is the primary process respiratory or metabolic

Step 3 is the an anion gap present?

Step 4 compensation formulas  pCo2=1.5(HCO3) +8      pCO2=0.75(HCO3) +20

Step 5 Use other data to help finalize the diagnosis (osmolal gap, lactate level, renal function)

Ted comment: there are only 3 endogenous ways for a patient to develop anion gap acidosis: ketoacidosis, lactatic acidosis, or uremia.  All the other causes of metabolic acidosis are exogenous.

We then worked through multiple acid-base problems.

 

Conference Notes 1-8-2014

New Year, New day of the week, so I am trying a slightly new format.  Based on the theory  “less is more”, I am going to try to make these a little shorter /more sustainable for me.  We’ll see how it goes.

Erickson/Schwab                       Oral Boards

I missed this excellent presentation , but the cases were

14 yo male withTesticular torsion.    Management was testicular u/s, pain control,  you can attempt  detorsion if the patient is having severe pain or the surgeon is delayed (Visualize opening a book.  Testicles are detorsed by rotating medial to lateral), and surgical repair is the definitive management.  

U/S of testicular torsion on left side

46 yo male with Lis franc foot injury.   If fx of the proximal second metatarsal is present or the middle cuneiform is not lining up perfectly with the second metatarsal you very likely have a lis franc injury present.  Also look at relationship between first metatarsal and first cuneiform

Lis Franc Fx/Dlx

65yo female  with  NSTEMI and flash pulmonary edema.    Treat with IV ntg and po ASA.   IV Lasix is second line additive treatment.

Sarah Maciolek          PED  APN Update

Sarah’s number if you have any pediatric patient safety issues you would want to bring forward  is 684-1782.   There are quarterly in situ pediatric simulations in the PED that include nurses, residents, and attendings.

Ketaneh/Lovell          Social Media

Largest HIPAA violation ever identified  happened on Facebook when a patient that frequented ACMC ED had a specific Facebook page that was created by a nurse from another hospital  about him.   This page was viewed by over 600 healthcare providers.  

Another example was a doc that was fired and fined for putting info about a trauma patient on Facebook.

28% of EM residency programs search facebook to get info on their applicants.

Any info you place on social media has to be general enough that the patient cannot be identified by the info in the post.

Don’t Friend patients on Facebook.   Don’t Friend people who in your workplace are in a subordinate position to you. 

Assume any text or post is publicly available forever.    Tweets are permanent.

Advocate Social Media Policy: HIPPA violations can lead to termination and legal action.  An unauthorized individual  can’t personally use Advocate’s Logo.   No ethnic slurs/personal insults/obscenity/harassment/inflammatory remarks allowed.  Advocate reserves the right to check an associate’s online profiles and content.

Beckemeyer question: Is it ok to text a  picture of a patient’s injury from the ED to a consultant.   Elise response: It is ok if the patient’s identity is protected.  Don’t put full facial view in the texted picture.  Also don’t give other identifying info in that text such as name or MR#.   When transmitting EKG’s be sure photo doesn’t include patient identifiers.    Girzadas comment: Be sure you document in the chart that the patient gave you permission to text the picture.     Elise comment: Even innocuous text or blog streams can inadvertently give identifying patient info.

Nick Ketaneh       Free and Open Access Medicine (FOAM) was discussed along with the use of Facebook and Twitter as sources of medical information  #FOAMed

Beckemeyer       Trauma in Pregnancy

Main Point:  Maternal resuscitation is the key to fetal resuscitation.

30 yo patient with gsw’s to head.   HR=120  BP=60/40

Physiologic  changes in pregnancy: HR increased, respiratory alkalosis, dilutional anemia, risk of IVC compression from uterus, increased pelvic vascularity making pregnant patients at risk for exsanguination from pelvic fx.   Increased risk of failed intubation.  GE sphincter is insufficient increasing risk of aspiration.  Increased minute ventilation and tidal volume but less functional residual capacity.  This makes the pregnant patient have less respiratory reserve.  Decreased chest compliance making bagging more difficult.

 IV access is better above the diaphragm in pregnant patients.  Access above diaphragm avoids the IVC compression problem with femoral lines or lower extremity IO access.

 

IVC Compression by fetus

Omi comment: Hypertonic saline was given to reduce cerebral edema and also avoid volume depletion from mannitol.    Hypotension could be due to bullet wound to head.  She has seen pt’s bleed severely from GSW’s  to head.  If bleeding is significant, suture/staple wound closed to tamponade bleeding.

Quick check for viability in the pregnant trauma patient:  Check if fundus is above umbilicus.  This grossly corresponds to 24 week gestation.   

The in-house phone number for the OB attending is 41-2005.

 Perimortem C-section: Start the procedure within 3 minutes of maternal arrest.  The procedure may aid maternal resuscitation by removing compression of IVC.   This procedure is best thought of as a resuscitative intervention for the mother and secondarily potentially life-saving for the child as well.    Toerne comment: This is a decision that has to be made rapidly.   If you delay, the window for success closes fast. 

Don’t stop CPR during the procedure.  Energy dosage for defibrillation is the same as in non-pregnant patients. Make a vertical midline incision, expose uterus , and make a vertical incision in the uterus.  Use scissors to extend uterine incision (Don’t cut the baby). Delivery baby and clamp cord.  Get procedure done within 1-2 minutes.   1 study showed no fetal survival if no fetal heart tones identified pre-delivery.   However lack of fetal heart tones should not impact your decision to perform perimortem c-section.  You should still do it for a fetus around 24 weeks as part of the resuscitation of the mom.  

Harwood comment:  Epi is a class C drug due to decreased blood flow to placenta.  You have to balance the risk between the fetus and the mom.  You could consider ED thoracotomy to perform open cardiac massage to improve cardiac output over external CPR.   Omi response:  Give epi in a pregnant patient with asystole or PEA.   The risk/benefit ratio favors giving it.  She voice a little discomfort with doing an open thoracotomy for cardiac massage if there is no specific intra-thoracic  injury to surgically repair.

Per ACOG, <5 rad there is no increase in childhood cancer.    CT abdomen gives 3.5 rad.

Lesser injured moms/fetus with trauma due to mvc or fall are monitored for 6-8 hours for contractions.   Harwood comment: A recent large study showed 4 hours was an adequate time period of fetal monitoring if no contractions or adverse fetal cardiac activity was noted in that time period.

Critical Care Equipment Lab